Per Com

You might also like

Download as pdf or txt
Download as pdf or txt
You are on page 1of 46

Chapter-S: Permutations and Combinations 281

Chapter-S: Permutations, and Combinations (n-l)! (n-2).1Ql)ln -l)1 _ 5


'" (n-4)! A (n + l)n (n' 1)! 12

t.
L#
r?r Cnd 2hr or,w:E
1i;'nvatua.t. "Pr,
:Jl4 : 156 or, 12n2 - 50n * 72:5n2 + 5n
sorution: ,rp, = L3rl?:Il!
or,'7n2 - 65n + 72: o
,q l7l
''Pn=ffi =- l'7.16.15. 14.13!
=57120 or, Trf - 56n - 9n + 72: 0

or, 7n(n - 8) - 9 (n - 8) : 0
'h,:pffi:at#t':6840 (n- 8) (7n- 9):0
1. (ii) If 4 x nP3 = 5 x *rP3 then lind the value of n. -
.'. n - 8, tr:1
9.Vutn:i9.is not accePtable
[cB'o5l
Solution:4xTr:5 xo-rPr 2-r
7 2
because then Pr='P, is undefined.
, n! :_. (n-l)!
ot,a1T4 )G;m '

4n.(n-l)! -(n-l)! Ans: n:8


or,@!::[I[ l. (v) Prove that, n-'P, + r.n-'P.-, - nP,

4n n-'P, + r. n-lPr-r
Solution:
or......-:)
'n-J
or, 4n:5n- 15 ' _ (n-l\! ,_ (n-1)!
-(n- I -r)!'''(n- I -r+ l)!
'

or, 15:5n -4n (n l)! (n- l)!


n: :-*_l_r -
.'. 15
nPo n-2P3
(n-r-l)!'"(n-r)!
t. (iii) If = 14 x then find the value of n.
_ (n-l)! _._ (n-l)!
IKUET' 05-061 -(n-r- 1)!''' (n-r) (n -r- l)!
sorution: np4: r4.*2p3. (n- l)! (. r \
n! . (n-2)!
or,(!_7r:14.66 -(n-r-l)! \'-r-t/
(n-l)! n-r*r
- n(n- l)(p-2)! :14
or,!ffiTf . . (n-2)! -(n-r-l)!' n-r
(n-T -rr

:- (n- l)! n
(n-r-l)!"n-r
or,n'-n-l4n-56 : --J!- - nprr
or, n'- l5n + 56:0
(n-r)!
or,rn'-8n-7n+56=0 1. (vi) Prove that the product of the first n odd

or,n(n-8)- 7(n-8):0
or, (n - 8) (n -7):O
numbers can be expressed in the form ffii
n: 7, 8
.'. tcB. lu
l. (iv) if *'P, , n*'P, = 5 z l2then find tlie vall:f Solution: 1.3.5........ (2n - l)
i. .4.5.6 ...;....... (2rr l\.2n
tRB.osl _ 1.2,.3

,Solution:
: t ,l.?.,*
n-'P, t o*tP,
.i
2.4.6...........,...2n

^- (n-l)! . (g|lv :5 :12 _ (2n)!


or,
ffi ' fff,'fr
- 2.1.2.2.2.3.......... 2.n
(n-l)! (n+1).n.(n-1)! 5 _ (2n)!. _ (2n)!
'', 1n+;! '(n-2Xn-3Xn-a)! - 12 - 2"(1.2.3..........n) - 2n.n!
282 Chapter-S : Permutations and Combinations

2. Find the number of arrangements of the letters Here 2520 x 2:5040


of the following words tanen all at * time: So the leffers cf the word 'CALCUTTA' can be
(i) laughter (ii) millennium arranged in twice as many ways as the letters of the
(iil) cricket :(iv) immediale word 'AMERICA' can be arranged.
(v) chittagong (vi) gqnmithe 3. (ii1 Strow that the permutations of the letters of
Solution: (i) The word 'laughter' consists of 8 the word 'RAJSHAHI' taken all at a time is four
leffers all of which are different. times the number of permutations of the letters
.'. Total number ofarranger.nt : 8! : 4A320 of the word BARISAL taken all at a time.
(ii) The word 'millennium' consists of l0 letters of [DB. 08, RB. 12, 08, BB. 06, MB. 10, 061
which 2 are'm's, 2 are '1.'s, 2 'i's, 2 'n's and Solution: The word 'BARISAL' consists of 7
remaining are different. letters of which 2 are 'A's and remaining are
different.
.'. Total number of arrangemelts :# 7l
:
= 226800
.'. Total number of arrangements
T. = 2520.
(iii) The word'cricket' consists of 7 letters of which The word 'RAJSHAHI' consists of 8 letters of
2 are 'e's and remaining are differenl- which 2 are 'A's, 2 'H's and remaining are
different.
.'. Total number of arrangements :# :2SZO
.'. Toal number of arrangements : 10080.
(iv) The word 'immediate' consists of 9 letters of #.:
which 2 are'i's, 2 'm's, 2 'e's and remaining are Here 10080:4x2520
different. So the permutations of the word 'RAJSHAHI' is
four times that of 'BARISAL'.
.'. Total number of arrangements: :45360. 3. (iii) Show that the letters of the word
h
(v) The word 'chittagong' consists of l0 letters of BANDARBAN can be arranged 2l times as
which 2 arc 't'S, 2 'g's apd remaining are different. many ways as the letters of the word SYLHET
can be arranged. uB. 171
.'. Total number of arrangements'j
# = 907200. Solution: The word'SYLHET'consists of 6 letters.
(vi) The word 'committee' consists of 9 letters of .'. The number of permutations of all the leffers is
which 2 arc 'm's, 2't's,2 'e's ,na turtuining are
:61 :720
different' Again, the word 'BANDARBAN' consists of 9
oi letters of which 3 are'A's, 2 are 'B's and 2 are N's.
:
,t;nl = 45360.
.'. Total number of arrangements
.'. The number of permutations of all the leffers is
3. (r) Show that the letters of the word :=+:21
CALCUTTA can be arranged in twice as many 3t2t2t t L ' x72O

ways as the letters of the word AMERICA can .'. The number of permutations of all the letters of
be arranged. [RB. 13, DB" 04r.MB. 12, ogl second place is 2l times the number of
Solution: The word 'CALCUTTA' consists of 8 permutations of all the letters of first place.
letters of which 2 arc 'C's, 2 'A's, 2 'T's and 3. (iv) Show that the permutations of the letters of
remaining are different. the last word of the name 'rlssac Newtont' taken
all at a time is 12 times that of first word.
.'. Total numberofarrangements :
#:5040. lcB. 171

The word 'AMERICA' codsists of 7 letters of ' Solution: I st word 'Issac' consists of 5 letters of.
which 2 are 'A's, and remaining are different. which 2 are s.

.'. Total number ofarrangements : : 2520. .'. Number of permutations i, : tf : oo


#
Chapter-S : Permutations and Combinations 283

t ast *ora'Newton'consists of 6 letters. Considering 3 vowels as I letter, number of letters


.'. Number of permutations is:6! : 120 = 12 x 60 is 6 and can be arranged in = 6! = 720 ways.
: 12 x p€rmutation of lst word Again 3 vowels can be arranged among themselves
.'. Number of permutations of the last word is 12 in3!=6ways.
times that of first word. :
.'. The number of arangements that vowels occur
[Here, N and n are considered as different letter in consecutive positions is = 720x6=4320.
since one is capital letter and other is small letterl .'. The number of arrangements that vowels are not
3. (v) There are 8 letters of which a certain number in consecutive positions is = 403204320:36000.
are alike and the rest are all different: if 6720 4. (iii) Find how many arrangements can be made
words can be formed with them taking all with the letters of the word 'IVIATHEMATICS'
together, how many letters are alike?
and in how many of them the vowels occur
Solution: Let x letters are alike.
together? UB. 14,12,01; BB. 14; CtgB. 14;
By the condition, : 6720 DB. 06, SB. 14, 04, 01, RB. 09, 06, CB. 12, 02,
}|
8! MB. ll, 07, 05, 031
or, x! :m
40320
or, x! =m Solution: The word 'MATHEMATICS' consists of
or, x! :6 or, x! :3! .'. x:3 ll letters of which 2 are'M's, 2'A's,2 'T's and
4. (t) Find the number of arrangenrents of the remaining are different.
letters of the word 'daughter' in which the .'. Total number of arrangement =#.
vowels are neyer separated, kr how many
arrangements the vowels do not occur in :4989600.
consecutive positions. [CtgB. 10, BB. 031 This word consists of 4 vowels which are A, E, A, I.
Solution: The word 'daughter' consists of 8 letters of Considering 4 vowels as I letter, total number of
which all are different.
It consists of 3 viweh. These are a, u, e
leffers is 8 and way of arrangement tr:# : 10080
Considering vowels as I letter total lettcrs is 6 and Again the vowels can be arranged among
arangement is : 6l :720 Ll
themselves in : 12 ways.
Again the arrangements among vowels themselves 7
:3! :6 .'. Total number of arrangement where the vowels
.'.Total number of arangements where the vowels occurtogether is: 10080 x 12:120960
are never separated is = 720 x 6= 4320. 4. (iv) Find the number of arangements that can
Total number of arrangements of ttp letters of the be made of the letters of the word
word'daughter' is = 8! = 40320, 6NEWZEALAND' so that the vowels may not be "
The number of ilrangements,where' the vowels in consecutive positions in any of them. tDjB. l7l
donot occur in consecutive positions is Solution: The word NEWZEALAND' consists of
:40320 10 letters of which 2 are'E' s, 2 are 'A' s, 2 are 'N' s
-4320:36000.
4. (ii) Find the number of arrangements that can be and remaining are different.
made of the letters of the word 6TRIANGLE' so
.'. Total number of permutations :#:453600
that the vowels may not be in consecutive
positions in any of them. This word consists of4 vowels which are E, E, A, A.
[BB. 10, DB. 0sr-ctg8.,07, MB. 0g] Considering 4 vowels as I letter, total number of
Solution: The word 'TRIANGLE' consists of 8 lettersis 7 andthe number of arrangement is
letters of all are different 1l
.'. Total number of arangement :8! : 40320.
=;__ 2s20
284 Chapter-5 : Permutations and Combinations

Again the vowels caR.,,be arranged among 4. (vii) Find the number of different arrangements
of the letters of the woril 'Chittagong' where all
themselve rinfi: 6uays. :
:

the vowels will remain together. [CtgB.0ll


.'. Total number of arrangement where the vowels
Solution: The word 'Chittagong' consists of l0
' occur together is = 252A x6 = t5l'S letters of which 2 are't's,2 'g's and remaining are
The number of arrangeilGrits:where the vowels different.
o.
i

do not occur in cons€cutive poSititinS is It consists 3 vowels which are i, a,


: 453600 -'l5l2}= 438480 Considering vowels as I letter, number of letters is
4. (v) Find the number of aiiafigeinents that can be
:
made of the letters of itre word {MUJIBNAGAR'
8 and can be arranged i, : # 10080 ways.

' so that the vowels may 'dof bd iii coiisecutive Again, the . vowels can be arranged among
positions in any of theriij
.. .i ;{*
lBB. 171 themselves in 3! :6 ways.
Solution: The word'nfi)JlBNA6"ARt'consists of I 0
.'. The number of arangements that all the vowels
letters of which 4 are vowefs'(J', I, A, A) and
remain together is: 10080x6 : 60480.
among vowels 2 are'A's.
4. (viii) How many words can be formed using the
.'. Total number of arrangoper* : = 1814400 letters of the word 'Technology' where the
#
Considering 4 vowels as l,.letter, tqtal number of vowels will remain together? [RUET. 05-061
letters is 7 and the number of arrangement is Solution: The word 'Technology' consists of l0 letters
=7t:5040 of which (e, o, o) 3 are vowels and (t c, h, n, l, g, y)7 ue
Again the vowels can be aifanged among consonants.

themselves q
4t
ways Considering the vowels as I letter, total number
r: 12
:
of letters is = 10 - 3 + I 8 and can be arranged
.'. Total number of arrangement whele the vowels
in 8! ways.
occur together is : 5040 x 12:60480
Again the vowels can be arranged among
The number of ur.ungrrrunts Wheri the vowels ?t
do not occur in consecutive pwitions is themselves in fr ways.
:
11814400*S480 = 1753920 ?t
4. (vi) Find the number o$iarrarypmcnts of the .'. Required number of words is: 8! xfi. : 120960

, Ietters of the word 'DEPRfSSilO{lt' in xhich the 4. (ix) In how many ways the letters of the word
vowels will remaiu together. ; i 1.'ni,, [CtgB. 17]
'r:
'arrange' be arranged so that the two 'r's will
Solution: The word'DEFRESSI@{-1 ,wrsists of l0 not be side by side?
letters of which 4 are vowels(,E, [:*,O).
Solution: The word 'arrange' consists of 7 letters 2
Considering 4 as I letsr,,bhl ,'tnrnbel of
-vowels ,. . are 'r's, 2'{s and remaining,arc 0ife11t
"-:
anangem*
.71
ir7
:

1l
letters is 7 and the number of
.'. The number of arrangements is:51. : 1260
Again the vowels'can be"''aiianged among
Considering 2.'r' as I letter, number of letters is 6
themselveri.fi*ayi, ". ,.-
, and can be arranged in =360 ways.
*
,

.'. Total number of arrangenlcnt'Where.the vowels


.'. The number of arrangements tt:,at 2 'r' will not
, occur together i, =**fi:30240 :
remain side by side is 1260 - 360 = 900
Chapter-5 : Permutations and Combinations 285

4. (x) In.how many ways may 3 grffn, 5 blue and 4 4. (xii) Seven letters A, B, C, D, E, F, G'need to be
red counters be arrangd in'a,iltw? How many arranged in such a way that A and B may not be
arrangements will have' trvo'tbiie c;btnters side in consecutive positions in any of them. How
by side? many arrangements are 'possible with such
Solution: Toal number ofcounters'is: 5 + 3 + 4: 12
condition? [BUET: t8-191
Solution: The number of pennutations of the given
of which 5 are blue, 3 green alrd 4"red.
7 letters is : 7!
.'. Total nurnber of arrangement *= # = 27720. Again, considering A and B as I letter, total number
of letters is 6 and number of permutations is : 6!
Again considering 5 blue counters as l, total
Again A and B can be arranged arnong themselves
number of counters is 8 and can be,arranged in
et
in 2! ways.

i*- = 280 ways. .'. Total number of arrangements where A and B


5 blue counters can be arranged,crong'ttremselves
will occur together is : 6! x 2!
<t .'. Required number of permutations :71 - 6r. x 2t
,n3r : I way.
J.

:3600
-

f. (i) In how many ways can the letter of the word


The number of ways that bluE counters will
.'.

remain side by side is:280x1:,28&. 'article' be arranged so that the vowels may
occupy the even positions?
4. (xi) Find the number of arranggments that
Solution: The word 'article' consists of 7 letters of
can be made of the letters,,sf,,,the word which 3 are vowels and 4 consonants.
'EXAMINATION' so that' ,thc cohsonants The positions that require to fill are shown below:
may not be in consecutive positions in any of o@@@o@@
them. [BUET: 17-181 3 even positions can be filled by 3 vowels in
3P, :6
Solution: The word 'EXAMINATION' consists of ways.
1l letters of whioh 5 are consonants (X, M, T, N, N) Remaining 4 positions can be filled by 4 consonants
aPa:24 ways.
out of these 2 are N's. Thi word consists 6 vowels in
(8, A, A, I, I, O) out of which 2,are A's and 2 are .'. Required number of arangement is :6 x24: 144

I's. f,. (ii) In how many ways can the letteis of the word
'equation' be arranged so that the consonants
:

.'.Toal number of arangement is


=ffi may occupy the odd places. uB. 081
=-4989600 Solution: The word 'equation' consists of 8 letters
Again, considering 5 consonants as 1 letter, total of rvhich 5 are vowels and 3 consonants.
number of letters is 7 and the number of The positions that require to fill are shown below:

:
1l ooo@o@@@
arrangement is
ffi:1260 4 odd positions can be filled by 3 consonants in
Again the consonants can .be au-apggd: ornoog
oP, : 24 ways.
5l Remaining 5 positions can be filled by 5 vowels in
tP,
=ways.
120

Total number of arrangements where the .'. The required number of permutations is

consonants occur together is = 1260 x 60 = 75600 =24x120=2880 :

t. (iii) In how many ways can the letter of the wdrd


The number of arangemerrts lrhrrr the vowels
'postage' be arranged so that the vowels maly
do not occur in consecutive'positions,is
occupy the even places? In how many do the
:4g8g600,= 75600 = abt4000
consonants occur together? lcB. 141
286 Chapter-S : Permutations and Combinations

Solution: The word 'Postage' consists of 7 letters f. (v) Find the total number of arrangements that
of which 3 are vowels and4 consonants. can be made with the letters of the word
The positions that require to fill are shown below: 'Textile'. In how many of these the vowels will
o@ @@ o @ @ remain together? In how many of these will the
vowels occupy the even places? [BUTEX: 02-031
3P,
3 even positions can be filled by 3 vowels in = 3r
:6 Solution: The word 'Textile' consists of 7 letters of
ways
which 2 ars't's, 2 'e's and remaining are different.
After filling 3 even positions by 3 vowels, the
number of ways that rernaining 4 positions can be "'. Total number of arrangements ir=fi.: 1260

filled by 4 consonants is aPa = 4, = 24 Considering vowels (e, i, e) as I letter, total number


.'. The number of ways that vowels occupy even of letters is = 7 - 3+I :5, among which 2 are't's
positionsis=6 x24=144
and number of arrangements ,r:ii
Again considering the consonants as I letter, total
number of letters is 4 and can be arranged in Again 3 vowels can be arranged among themselves
4t :24 ways. .
,n
3!
T. ways.
But the consonants can be arranged among
The number of arangements that the vowels
themselves in 4! =24 ways.
The number of affangements that consonants occur remain together t,: # * ]f : rto
together is =24 xV|=576. This word consists of 3 vowels (e, i, e) and 4
5. (iv) How many different permutations can be consonants (L L x, c).

made out of the letters of the word The positions that require to fill are shown below:

ALLAHABAD, each of the nine letters being I, II, III, IV, V, VI, VU

used only once in each permutations? In how 3 evenpositions can be filled by 3 vowe* *||ways.
many of these permutations will 4 'A's be
Remaining 4 odd positions can be filled by 4
tegether? In how many of these will the vowels . 41,
consonants rn ways.
ftcmpy the cvcn places? lrvlB. 011 ,I
Solution: The word 'ALLAHABAD' consists of 9
letters of which 4 are 'A's, 2'L's and rernainiiig are
So required number of arrangements i, =J| ,fi = x
different. t. (vi) How many numbers of seven digits can be
formed with the digits 3, 41 3) 4, 5r 61 5 so that the
Total number of permutations iu = : 7560
.'.
#i odd digits always occupy the odd places?
Considering 4 'A's as 1 letter, number of letters is 6 Solution: Given digits 3, 4, 3, 4, 5, 6, 5.
4 digits are odd which are 3, 3, 5, 5; where two 3's
and can be arrangeo * ti: 360 ways. and two 5's.

Again 4 'A' can be arranged among themselves in And 3 digits are even which are 4,4, 6; where two
4's.
4t
Ii= l ways. The positions that required to fill are shown below:

.'. The number of permutations that 4 'A's remain


o@ @@ o @ @
4 odd positions can be filled by 4 odd digits in
togetheris:360x1=360 AI
:6
4 even positions can be filled by 4 'A's in I way fr. ways.
and 5 odd positions can be filled by 5 consonants . Remaining 3 even places can be filled by 3 even
?t
(L, L, H, B, D) in
il : uo ways. digits infi :3 ways.

Ans: 7560,360,60 .'.Total nurnbers :6 x3 : 18


Chapter-S : Permutations and Combinations 287

6. Find the number of ways'in whieh the letters of E. (ii) In how many ways can the letters of the word
the word (PERIT{UTATION' mai be rearranged daughter be arranged? How many arangements
without changing the pbsiiiofst8fany vowel? begin with r and end with d?
Solution: The word "PE i1q.tl6{u.'ionsists of Solution: The word 'daughter' consists of 8 letters
I 1 letters of which 2"are'"i'i.''li' 6.ifirists'5'vowels
. ' "
'{.'{'
all of which are different.
.'. Total nurnber of arrangement is =
8Ps :40320
Without changing the' posillo, ;f,.rry vowel that is
1:. ,
Starting with 'r' and ending with 'd' remaining 6
n be arranged * :$ :360 ways. letters in 6 places can be aranged :6Po :720 ways

is (iii) How many different


'

The word 'PERMUTATI . 'u,1lym


i: I

an 8. arrangements can be
arrangement. made of all the letters of the word 'engineering'?
, ,. . :_ ,, l
In how many of them the three 'e's stand
.'. The number of rearranggmefi! I :359
;l9O together and in how many wilt they stand first?
7. Find the number of ways in w$q!r,.t-be letters of
the word 'director' may.be f€qrraEg$ , Solution: The word 'engineering' consisb of I I letters
(i) without changing the order,,qf the,yervels of which 3 are 'e's, 3 'n's, 2 'g's,2'i's and remainlng
(ii) without changing the place gfany-,vgyel are different.
(iii) without changing the relative posithns of the
Total number of arrangements t, :
vowels and the consonants? , [BIf. m-0tl .'.
#
Solution: The word 'Director' o 4glsts of 8 letters =277200.
of which 2 are'r's and remaining are different. Considering 3 'e' as I letter, total number of letters
It consists of 3 vowels and 5 consgnants,
is 9 and can be arranged ,, =
#: l5 120 ways.
(i) Since the vowels will remain in Egng order, let
us consider that they are same:lejt€rsii],:, : Again keeping 3 'e' in first, remaining 8 letters can
So out of 8 letters 3 are same and 2 'r's.
be arranged in :#iri = t68o ways.
... The number of arrangernents is :3|+'= 3360
, ". -1.._- Ans: 277200, 15120, 1680
(ii) Without changing the position of any vowel, that is 8. (iv) In how many ways can the letters of the
only consonants can be arranged ti = 60 ways. word 'identity' be arranged? Hbw many will
+ have fi' at the beginning and 'i' at the end? In
(iii) It consists of 3 vowels.,g1fl,5 gonsonants of
how many the two'i's and two't's be together?
which 2 are 'r's.
Solution: The word 'identiff' consists of 8 letters
3 vowels can be arranged amopg;thmsplves in
of which 2 are 'i's, 2 't's and remaining are
3!=6ways. ,'.. : ...,.
different.
Again 5 consonants : can , bG, t€.mpd: among
5f .'.Total number of arrangements tr: # = 10080
themselves in
fr: 60 ways. . :, :_
,:

.'.Required number of rrqernerfs ir=6rx60 : 360


Keeping 2 'i' at the beginning and at the end,
remaining 6 letters can be amanged in 6 positions in
8. (i) How many armngemc*tsicln:bc'qsdo out of
the letters of the wsrd'toqttatbn",tl#,of which =360 ways.
*
Again considering 2 'i' as I letter and 2 't' as
Solution: The word 'equationi?fiSsfsof 8 letters
'another I letter, total number of letters is 6 and can
all of which are different.': i ' r'" :r:

Keeping 'e' at the begiming, retnatilihg.T letters


be arranged in 6! = 720 ways.
can be arranged in 7 ptaces'=
7Pr'ti
Tl- 5040 ways
Ans: 10080,360,720
288

8. (v) How many arrangements can be made out of (ii) How many arrangements can be made out of
the letters of the wor{.'Calculus' all of which the letters of the word 'courage' all of which
have 6u' at the beginning and ,fhe 9nd? How have a vowel at the beginning?
many..arrangements begin and e1d with same Solution: The word 'courage' consists of 7 letters
letter? TBUTE>C0,6T: BIT. ece1l all of which are different. It has 4 vowels.
Solution: The word 'Calcultis' consisis of 8 letters Out of 4 vowels, 1 vowel can be arranged at the
beginning in :
aPr
of which 2 arc'c's,2 'l's and 2 'ut. = 4 ways.
Keeping 'u' at the beginning and the end, remaining Remaining 6 letters in 6 places can be arranged
: 6Po :720 ways
6 letters can be arranged h#: 180 ways.
.'. Total number of arrangements : 4 x 720 :2880
Similarly keeping 'c' at the beginning aqd the end, 9. (iii) Find the number of arrangements that can
remaining 6 letters can be,arranged ,"# ways.
be made of three letters from the letters
a, b, c, d, e, f so that each arrangement consists
Again keeping 'l' at the beginning and the end, of at least one vowel?
remaining 6 letters can be arranged khways. Solution: Among the letters a, b, c, d, e, f; 4 we
consonants and 2 vowels.
So required number of,arrangement is 6P3
Taking 3 letters from 6, words can be formed in
6! 6! 6t
=fr.* ways.
ZW*ffi.:540 Among them words consist of all 3 consonants is
aP3

8. (vi) How many arrangements can be made out of


.'. The number of words that consists of at least I
the letters of the word 'TESTICLE' all of which
vowel is:6Pr - oP, : 120 - 24:96
have not'E' at the beginning and the end?
. 9. (iv) How many different words can be formed
[sB. 171
from the letters of the word 6Computer' taken 3
Solution: The word "TESTICLF" consists of 8 letters
at a time which consist at least one vowel?
of which 2 are'T' s, 2 we'E' s and remaining are
different.
IRUET.0e-l0l
Solution: The word 'ComputeC consists of 8 letters of
.'. Total number of arrangernents =#r= 10080 which 5 are consonants (c, m, p, t, r) and 4 are vowels
(o, u, e).
Fixing 'E' at the beginning and at the end remaining
.'. The number of words formed by 3 letters taking
6 letters can be arranged in = = 360 ways. 8Pr:336
$ from 8 letters is =
So the number of arrangemsnts which-have not 'E' at Now the number of words formed by 3 consonants is
=sPr=60
the beginning and the end is = 10080 - 360 = 9720
9. (i) How many words can be formed with I vowel So the number of words which consist at least one
and 2 consonants form the leffers of the word vowel is:336 -60:276
9. (v) Using a digit only once how many numbers of
'second' so that the vowel may always occupy the
middle position? [BB. 0U three digits can be formed out of the digits 2, 3'
Solution: The word 'Secondn consists of 2 vowels 5r7 r8r 9 that consists at least one even digit?
and 4 consonants. Solution: Given digits 2,3, 5,7, 8,9. Among these
The words that are required to form'consist I vowel 4 digits are odd and 2 digits are even.
and 2 consonants where the vowel will occupy Number of permutations taking 3 out of 6 'is
middle position. - uPr : l2o'
I vowel from 2 can be arranged.in :2 ways
2P,
$Bain, number of
permutations taking 3 odd digits
2 consonants from 4 can be''dri'anged in 4Pz: 12 . out of 4 is=aPz:24
ways .'.Required total numbers that contain at least one
.'.Number of words :2 x 12:'24j even digit is = 120 - 24 :96
289

9., (vi) In how many ways can the,letter tlf the word 11. (i) Out of twenty six letters of the alphabet. in

'Mechanics' be arranged so thf th6 vowels and how many ways can a word be made consisting
consonants will be separately together? of 5 different letters, two of which must be a and z?
Solutibn: The word 'Mechanici"'torriists of 3 Solution: 5 letters out of 26 will be taken that must
vowels and 6 consonants. Among consonants 2 contain a and z.
are tc's. Firstly, 2 positions out of 5 can be filled by a and z
:

Considering vowels as 1 .letter and consoqants as


in 5Pr:20 ways.
another 1 letter, total numbq of le$ers is 2 and can Remaining 3 positions can be filled by remaining
be arranged in 2! :2 ways. :
24 letters in'oP, 12144 ways.
Again 3 vowels can be arranged'milong themselves
.'.Total number of words is:12144 x20:242880
-Dc'.. 11. (ii) In how many permutations of 10 things taken
And 6 consonants can anafi$ed iamong
:
5 at a time will2 particular things always occur?
themselve, in
$: 360 ways. lcB. 10, sB. 031

.'. The number of arraBgemgiits flrat'"fits'vowels Solution: Firstly, 2 positions out of 5 can be filled
5P2
and consonants will be separately'fogether is by 2 particular things in ways.
:2x6x360:4320 ' Remaining 3 positions can be filled by remaining 8

10.
:i.,
passwords of four symbols can be ma$e without t:tffiI":ffil*
8pr
repeatition using the 26 l-etter$ of English orpermutations is:5pz x
alphabets and 10 digits of Mathst&tics. :6720 '

(i) How many passwords can be me4e in. total? 11. (iii) How many arrangements can be made out of
(ii) How many of them will startrtrvilt,striEnglish the letters of the word 'COMPUTER' all of
letter? which have 6R' at the end but not 'C' at the
(ii) How many of them will start wlth an English beginning? IBIT. 01-021
letter and end with Mathematicsl dff Solution: The word'Computer' consists of 8 letters.
26+10::36
I symbot = Except at the beginning and the end, the leffer 'C'
Number of passwords taking 4 out of 36 symbols can be arranged in 6 places in
6P1
ways.
36Pr
is = Now keeping R at the end, remaining 6 letters can
(ii) At the beginning I letter out of 26 can be
in 6! Ways.
be arranged
26P1 :26
arranged in ways.
.'.Total number of arrangement - uP,
x 6! : 4320
3 more symbols out of 35 can be aranged in
11. (iv) Words consisting of 4 letters are formed "
. .=rfprways.
from the letters of the word 'equation'. How
.'. Total number of password is :5x "P,
many of the words contain tn' but not tet?
(iiD At the beginning I letfe.r out,sf,,26 can be
-'i"' Solution: The word 'equation' consists of 8 letters
= 26 ways.
26P1
arranged in
all of which are different.
At the end ldigit out of 10 eantE'Aii$dbbtl'in 'h,
' In 4 places the letter 'n' can be aranged aP, : 4
1
=I0 ways.
a
!c 2 more symbols out of 34 can be anangid in ways.
d
ai . .:
, :,
,ipr*uy, 3 more letters out of remaining 6 letters can be
d .'. Total number of password is. arranged:6Pr : l2}ways
$
.; =26 x l0 x 3aP3* 260 x3aP2 .'. Required number of words = 4 x 120 = 480
o
il i
290 Chapter-S : Permutations and Combinations

I l. ' (v) If the number of permutations of n different 13. (i) trr how many ways can 14 I.Sc. students and
things taken r (n > r) at a time in which one l0 B.Sc. students be arranged in a line so that nc
particular thing alw*ys (rccurs is. equal to the two B.Sc. students may occupy consecutivt
'number of permutatio.ns in wlieh that thing positions? lJB.04l
does
Solution: 14 students of I. Sc class can be arangec
not occur be equal then show that, n =2r.
in a line in 14! ways.
Solution: Out of ir' positiomr 1 position can be
In this line of 14 students there are 13 positionr
filled by 1 particular thing in'P1 = r ways. between them. At the beginning and at the end thert
Remaining (r - l) positionq can be filled by (n - 1) are2 more positions.
things in
n-rPr-1
ways. These (13 + 2): 15 positions can be filled by l(
l5P1s
students of B. Sc class in ways.
.'.Number of permutations: r. *''P;-, =..ffi .'. Total number of arrangements = l4t r "P,o
Again, excluding t
paiticular thing i.e. using 13. (ii) The executive committee of a club consists o1
(n - l) things 'r' porition, can be filled in n-rP, or, 2l members of which 8 are females and 13
(n - l)!
males. Excluding 2 male members, in how man)
(n-r-tX waYs', "' ways the remaining members can be arranged in

(g- l)['_ '(n- 1)!


a line so that two female members will no1
By the condition, r.
(n-r)l (n-r- l)! occupy consecutive positions? [CtgB. 17]
Solution: Excluding 2 male members remaining
r __
or,m:G-;_t) "1
male members is (13 - 2): ll

orrr:n-r I I will be arranged in a line so tha


male members
two fernale members will not occupy consecutive
." n:2t
positions.
12. (i) A library has 8 copies of one book, 3 copies of
each two books, 5 coiiies each of three books and
In this line of I I male there are l0 positions
between them. At the beginning and at the end thert
single copy. of 10 books. In hrinp'many ways can
are2 more positions.
atl the books be arrdnged?
These (10 + 2) : 12 positions can be filled by t
Solution: Total books :8+2 x 3 +3 x5+ l0 x I :39 r2Ps
females in ways.
.'. Total number ofarrangenrern, = Again, I I males can be arranged in that line in I
tr lr#i-sr 1 l

sr
ways.
.'. Required number of arrangements is : t2Ps
x l1!
t2. (ii) In how many ways 'can 2 books on 14. (r) 5 students will stand in cyclic order. In hou
Mathematics, 5 books on,Physics and 3 books on many ways they can take position?
Chemistry be arra4g€d ee,a shelf so that books Solution: In cyclic permutation one position neec
on the same subject may be together? to be fixed.
Solution: Considering 2 books oa,Mathematics as I .'. Keeping position fixed, remaining (5 - 1) : z
I
Broup, 5 books on Physics as another group and 3 students can take position in 4! : 24 ways.
books of Chemistry as thiid grotip; 3 groups can be So total number of ways is24. :

arranged in 3! :6 ways; t4 (ii) In how many ways can 8 members of a fami\


Again, sit at a round dining table of 8 seats?
2 bools on Mathematics can be arranged in 2l - 2 ways. Solutidn: In cyclic permutation one position neet
5 books on Physics canbe grynggd,in 5! = 120 ways. to be fixed.
3 books on Chemistry can be mnged in 3! = 6 ways. .'. Keeping I position fixed, remaining (8 - t; = '
.'. Total number of arrangements : 6xZxl20x6 students can take position in 7! : 5040.ways.
' - E640 So total number of ways is 5040.
Chapter-5 : Permutations and Combinations 291

14. liii; tn how many ways can 5 science students Solution: Given digits consist of 4 even digits and
and 5 Arts students sit at a round table so that 5 odd digits.
no tw9 Arts students may be side by side? At the beginning and at the end positions can be
lDB. 12, BB. lU filled by 4 even digits in aP2
= 12 ways.
Solution: Fixing I student of, science, remaining 4 Remaining 7 positions can be filled by remainingT
can be arranged in 4! : 24 ways. digits in
7P7 :71 :5040 ways.
There are 5 positions between'these 5 science .'. Total numbers : 5040 x 12:60480
students that can be filled by 5 arts students in 15. (iv) How many different numbers of six digits
'Pr:5! : l2o ways. may be made with the digits 3, 3, 1, lr l,4? How
.'. Total number of arrangements = 24 x 120:2880. many of them are greater than 400000?
14. (iv) In how many ways can 12 different pearls be Solution: Given digits 3,3, l, l, 1,4
strung on a band so as to form e necklace?
:
Solution: Fixing I pearl, remaining I I pearls can .'.Number of total permutations
#r:ffi: uo

bearrangedin ll!ways. Here numbers greater than 400000 will start with 4
Again for clockwise and for anti-clockwise and of 6 digits.
permutations, same necklace is fcntned.
.'.Keeping 4 at the beginning, required number of
.'.Number of ways :+
15. (i) How many numbers of three digits can be
permutations is: #r:ffi: ro

Ans: 60, 10
formed with the digits 3, 4, 5, 6, 7, 8 without
I5. (v) How many numbers lying between 5000 and
repetition of any digit?
6000 can be formed with the digits 1,2, 5, 6,7,8
Solution: Given digits 3, 4, 5, 6, 7, 8
without repetition of any dig,it? lBB, 131
Number of 3 digit numbers formed using given 6
:120 Solution: Given digits 1,2, 5, 6,7 ,8
digits is :6Pr
Numbers between 5000 and 6000 are of 5 digits and
15. (ii) How many numbers of six digits can be
start with 5.
formed with the digits 3, 4, $
6, 7, 8 without
.'. Starting with 5, number of permutations taking 3
repetition of any digit? How many of these will
not be divisible by 5? more digits out of remaining 5 digits is =
5Pr : 60
Solution: Total digits - 6 15. (vi) How many numbers of six digits can be
.'. Total numbers formed by all 6 diglts = 6l = 720 formed with the digits 6, 2, 7, 0, 9, 8? In how
Numbers divisible by 5 must have 5 in the unit many of them 0 will occupy the unit place?
place. Solution: Given digits 6,2,'1,0, 9, 8
Keeping 5 in the unit place number of Number of permutations formed with all 6 digits is
: : - uPu :61':720
permutations by remaining 5 digits is 5! 120
.'. Total numbers that are not divisible by 5 is But numbers starting with 0 are not of 6 digits.

= li,o -'l2o : 6oo .'. Keeping 0 at the ftrst, taking remaining 5 digits
15. (iii)'Find the number of numbers which can be number of permutations is: 5t : 120

formed with the digits l, 3, 4, 5, 6, 7, 8, 9 .'. Total numbers of 6 digits is :720-120 :600
,2, '
using each digit only once"lii=.a number which Again, keeping 0 in the unit place number of
will have even digits a?the beginning and at the permutations with remaining 5 digits is :'sPs : 51: 120,.
end. .'. 120 numbers will have 0 in the unit place
292 Chapter-S : Permutations and Combinations

15. (vii) How many odd,'numbers of 5 significant Even numbers have even digits in unit place. So
digits can be formed with tlre digits of the required numbers will have 6 or 2 or 0 in the unit
number10652? . ,:: lDB. 171
place.
Solution: Given digits 1o 0,,6,. 5, 2
Keeping 6 in the unit place, number of permutations
Odd nurnbers have add 4€rJs"in,'unit place. So
required numbers will heve lor 5 in the unit place. with remaining 4 digits is = 4! = 24.
Keeping 5 in the unit place,-quqb.er of permutations Again numbers starting with 0 will not be of 5

with remaining 4 digits is:4!' :24. digits.


But numbers starting with 0 me not of S Cigrts.
.'.Keeping 6 in the unit place and 0 at the first,
Keeping 0 at tfie fust, niiniber of numbers
number of permutations with remaining 3 digits is
formed by remaining 3 digits 3! = 6.
3! :6.
So total 5 digits signifrcant odd ildnfei.s in this case
is:24 - 6: 18 . i_
'24-6: l8
.'. 5 digit numbers
Similarly, keeping 1 in the unit place number of 5 Similarly, keeping 2 in the unit place 5 digit.
digit significant odd numbers is: 'I8 numbers : l8
.'. Total odd numberi is = l8 + 1S: 36
Again, keeping 0 in the unit place 5 digit numbers
15. (viii) How many odd nurnbehs'of ,.5 significant : :24
41.
digits can be formed with the digits 6, 5,2,3, 0
.'. Total even numbers : 18 + 1 8 + 24: 60
using each digit only once in a number?
16. How many numbers greater than 1,001000 can be
lCB. 13, CtgB. 11,05, DjB. 1I, SB. 13, 10,07,
JB. 13, 09, BB. 09, RB. 07, DB. l4,ll,07,0ll formed with the digits I,3,0,3,5,5?
Solution: Given digits 6,5,2,3,0 Solution: Given 6 digits consists two '3's and

Odd numbers have odd digiB in'unit place. So two'5's.


required numbers will h,ave 3'oit 5 itr:the,unit place. :#.=
.'. Total number of permutations 180
Keeping 5 in the unit plabe, nurnU6r"of,p€rmutations
with remaining 4 digits is : 4! :24.
Numbers of permutations starting with 0 ir:1fr: :O
Again numbers starting with,;O wtll not be of 5

digits. .'.Numbers greater than 1,00,000 are : (180-30) = 150

.'.Keeping 5 in the,unit placeiamd.s at the first, 17. (i) How many numbers greater thau 4000 can be
number of permutations wt& rrmaifring 3.digits is formed with the digits 0, 3, 5, 6, 8, without
3! :6.
repetition of any digit in any number?
:24-6:18 : ,r:,;i .-'
.'. 5 digit numbers
Solution: Given digits 0, 3, 5, 6, 8
Similarly, keeping 3 in the urit -place 5 digit
Here 4 digit numbers that greater than 4000 must
numbers: l8
start with 5, 6 or 8.
.'. Total 5 digit odd numtters'i= 18 + I8:36
15. (ix) How many even numbercof$slgnificant digits Number ofpermutations : 3x
4Ps:
72

can be formed with the:digits'61,5i ,2r,3, 0 using Again, number of permutations of all 5 digits

each digit only oncein a nu#Br?,:.:,: :5! : 120


". ;t;.;,,,,f88. $8, RB. 071 But out of them 4! or 24 numbers start with 0.
. Solution: Given digits 615,.2,,3;Q ,
.'.} digit numbers = 120 -24:96
.'. Total nunibers = 72 + 96 = 168
Chapter-S : Permutations and Cornbinations 293

17. (ii) How many numbers of not more than three But starting with 0 and keeping 5 in the unit place
digits can be formed using the,digits 011,2,3,4 number of arrangements is :8Pr : 8
using a.digit only once? .'. Keeping 5 in the unit place total 3 digit numbers
:72 - 8: 64

Required numbers will be of I or 2 or 3 digits. Again, keeping 0 in the unit place, 2 more digits out
(a) Total I digit numbers is 4 formed by given of remaining 9 digits can be arranged in eP2 : 72

digits. ways.

(b) 2 digits out of 5 digits can be'irranged in .'.3 digits numbers that are divisible by 5 is in
total : 64 + 72: 136
.'.Total required numbers : (l + 17 + 136): 154
But numbers starting with 0 will no] be of 2 digits.
18. How many numbers of 3 digits can be formed
.'. Starting with 0, 1 more digit out of remaining 4
with the digits l, 2, 3, 4, 5 using a digit any
aP1
digits can be arranged in = 4 ways. number of times? How many of these numbers
.'. Total 2 digit numbers :2A - 4: 16. consist of two or more of the same digits?
(c) 3 digits out of 5 digits can be arranged in Solution: Given digits I , 2, 3 , 4, 5
tP, 3 digits can be arranged with repetition out of 5
= 60 ways
But numbers starting with 0 will not be of 3 digits. digits in 53 or 125 ways.
5P3
.'. Starting with 0, 2 more digit out of remaining 4 Without repetition 3 digits can be arranged in or

digits can be arranged in


aP2 : tr2 ways.
60 ways.

.'. Total 3 digit numbers : 60 - 12 = 48. Total numbers that contain two or more equal
: - 60:65
total numbers : 4 + l6 + 48 : 68
digits is 125
.'.Required
Ans: 125,65.
17. (iii) Find the number of numbe{s less than 1000
19. (i) The telephone dial is marked with the digits
, and divisible by 5 which can be formed with the from 0 to 9. How many telephone connections
digits 0rlr2r3;4,5, 617rs9 each digit occurring can be given in Dhaka city if the telephones of
not more than once in each number. Dhaka are of 7 digits? [S8.05, R8.09,20001
Solution: Required numbers will !qv-q 0 or 5 in the
Solution: Total digits = l0
unit place. Numbers can be of I or 2 or 3 digits.
.'. Number of permutations using 7 digits out of 10
(a) There is I number of 1 digit which is divisible :
digits with repetition is 107: 10000000
bv 5.
(b) Keeping 5 in the unit place, I more digit for the
So the number of telephone connections is
10000000
tenth place out of remaining 9 digits can be
arranged in
qP1
:9 ways. I9. (ii) The telephone dial is marked with the digits
But I number starts with 0. from 0 to 9. How Tany telephone connectionS
Keeping 5 in the unit place tobt 2 digit numbers can be given in Chittagong city if the telephones of

= 9 -l=8 Chittagong are of 7 digits and not start with 0?


Again, keeping 0 in the unit place, I more digit for Solution: Total digits : 10
the tenth place out of remaining 9 digits can be .'. Number of permutations using 7 digits out of 10
arranged in
eP1
- 9 ways. digits with repetitions is : 107: 10000000
.'.2 digits numbers that are divisible 5 is in total Again, fixing 0 at first number of permutations
using 6 digits out of l0 digits with repetitions is
(c) Keeping 5 in the unit piace, 2 msre digits out of : 106: 100000Q
i remaining 9 digits can be arranged in -.ePz : 72 Sothe number of telephone connections is
ways. : 10000000 - 1000000:9000000.
294 Chapter-S : Permutations and Combinations

t9. ,n. mobile numbers of Banglalink formed


t,rrl 20. A signaler has 6 flags of which one green, two
with l1 digits of which first three are code white and three yellow green flags. Find how
numher 019. How many mobilc connections can many different signals he can make (i) by using
be given with this condition?
exactly six flags (ii) by using exactly 5 flags?
Solution: Total mathematical digits = l0
Out of 1l digits, first 3 digits are fixed. tDjB. 10, cB. 011

.'.Remaining 8 positions can be filled using l0 Solution: (i) Out of 6 flags 2 are white and 3 are

digits with repetition in 108 ways, : , yellow.


Ans: 108
*'19.
(iv) In how many ways a Facebook E) password .'. Total number of signals with all6 flags :fi : OO
j; r of 6 symbols with repetitio:r can bq given using (ii) Here the probable selections of 5 flags are:
(a) English alphabets (b) Mathematical digits
(c) both?
Green White Yellow

Solution: (a) Total letters :26


(a)t 2 2

Number of permutations using 6 letters with


(b)l r 3

repetition is: (26)6 (c)0 2 3

(b) Total digits: l0 sl


(a) The number of signals :fi'r:30
Number of permutations using 6 digits rvith
repetition is: (10)6
(c) Total letters and digits : ()$+10) = 36
(b) The number of signals :# : ro
Number of permutations using 6 digits with
repefition is : (36)6 (c) The number of signals :ffi5l : l0
19. (v) A lock has 4 rings each marked with the
.'.Total number of signals using 5'flags is
digits from 0 to 9. If the lock'Eiln'be opened for
only one arrangement of 4 lettcr3."Iind how many =30+20+10:60
unsuccessful attempts chn be,'hlide'to'open the
lock fE*r.t..62.|
[RB. I4l
Solution: Total digits: 10 , : 1. (i) Evaluate: rsC11
,
toCou,
'o*Crr*
Number of permutations using,4 digits with
r-il
Solution: ".'-- ----JL
- l 1!(15-11)!
The lock can be opened for only one arrangement. l5.l4.t3.tz.ttt:-:32760 1365
.'. Required options: 104- I l1! 4! 24
19. (vi) There are 3 candidates for a pppfessorship
and one is to be elected by the votes of 5 men. In
*5on:A
v46
50.49.48.47.46! :230300.
46!(50-46)! 46t 4t
how many ways can the votes be given?
1 10004' 1000
[RB. 10, CB.'09, JB. 051 \,998 - e98!(1000-e98)!
Solution: Out of 3, one cardidate wiltr &slected.
1000x999x998! :499500
One candidate can get ufinost 5 rotes,.
9981 2t
.'. Votes can be given in 3s :2$
ways.
19. (vii).In how many waysi can,3lp,rizes, one for 1. (ii) If nCr: nC3
then 1ind nCs .

good conduct, one for sports ad one for general Solution: Given, nCz: oCl
proficiency be given away to-10,boys? :
,'. [ 7 + 3 ['.'
nC*
-'C, =) n : x + y]
Solution: Total number of boysE l0
One boy can get all 3 prizes.
+n: l0
.'. 3 prizes can be given to l0 boys with repetition . nrt l0!
rv8-
-ron - (lo-8)! -l0x9x8!
'"s-8! - 8! x2 -r.
-+r
in 103: 1000 ways.
Chapter-S : Permutations and Combinations 295

l. (iii) If nC, : nCn-,, then find nCp (r+ l)! (n-r- l)! _ I
.
--
urr -2
Solution: Given, nC, : nCn-
ro
r!(n-r)!
:) nCr. nC.
^- (r+ l)r! (n-r- l)! - I
nCn
= 3Cn =
' =+
nC,
= lCro
- (n - rol ['.' - r]'
orrm:,
:) n: 5 + l0 [...nC*: nC, n =x + yI r+ I I
= or,il:i. or,n-r=2r+2
n: 15
. n/^ l5n lJ! or,n- 3r:2........(i)
\.r2 - r-r2 - _ l2)!
12! (15 oCr*1 nCr,r2:2
Again, I :3
_ rs x 11 x 1?! _ ls , I,j I,l? :
lt: x3! qss
5 "::'" n!
12!
(iv) If "C, : ttC.*, then find 'Cs. lBUff:
(r+ l)! (n-r- l)! z
l. 98-99I orrT 3
Solution: ttC.- "C*, + r*r+'2 = l8 (n-r-2)!
(r+2)!
+2r=16 .'. r:8 t!

tCr:8Cs:56[ ^- (r+2)! (n-r-2)!


or,m:t -2
nP3
1. (v) If =2 xnCc then find fhe
^_ (r+2)(r+ l)! (n-r-2)! _2
:3
t7l orr
r+2 2
or'iET:J
.. n! :1v n!
o^41.(n-a)! r..,
-(n-3)!
t2 or,2n-2r-2:3r*6
or,2n - 5r: 8 .............. (iD
l1
* n-3- (ii)-(i)x2+
12
2n-5r-2n+6r:8-4
= n - 3 = 12.'. n = 15
l. (vi) If 'P, : 240 and 'b. = l2b"then find the .'. r: 4
values of n and r. '":- {Ctg. n. 1U From (i) we get, n : 3r + 2:3 x 4 + 2: 14

Solution: nP,= 240 or, :2#........,...(l) .'. n- l4,r:4


1$i
l. (vii) The number of (r +
Ancl
nC, : l20or,
ffi = lZN..-..............(2)
combinations
(r + 2) and (r + 3) things taken from 6n' different
,of
1),

(1)* Q)=rl:2or, r:2 , .. things respectively are in the ratio 15 : 24 : 28.


i.. :. ,i,:

In (l) puning r:2weg.r,# =.b4fr Find the value of n and r. [KUETi 19-201
'.-' nCr* nCr*
or, n(n-l ):240or, n'- n -''24'0* =:'0 Solution: ,: 2: 15 :24

or, (n-16) (n + 15) = 0 .'. n =.16r n,=L 1'5 n! n! l5


But the value of n cannot be negative. =(r* l)!(n - r -
.'. n- 16,r=2. n! __ (r+2)!(n-r-2)! _5
1. (vii) If nC, : nC*r 3
nC,*z
= 1 : 2 : 3then find the
=='>(r+ I,x(n-r- l)! ^ n! 8

values of n and r.
Solution: nCr:oCr*1 :l:2 '. ': 'j::
nL
r! (n - r)!
otrF =-L
2
(r+ l)! (n-r- 1)!
296 Chapter-5 : Permutations and Combinations
'Again, nCr+2 nCr+3
I :24'.28 , (iii) How .many triangles can be formed by
n! .-:- n! 24 joining the angular points of a plane figure of 12
- (r+2)!(n-r-2)!' (r+3)!(n-r-3)! 28 sides? How many diagonals it has?

n! lDjB. 11; JB. l0; CtgB.03l


(r+2)!(n -r-2)! Solution: A polygon of 12 sides has 12 angular
points.
I - r-
=@*T=7 G + 3Xr+ 2)!(T 3)! _ 6
From the 12 angular points, joining any 3 points
triangle can be formed.
a

r+3 6 + 6n 6r l2:7r +21


+..-......=:; So number of triangles formed by selecting 3 points
n-r-z - - r2Cr :220
out of 12 points is: ways
.'. 6n - l3r: 33 ........ ...,....... (ii) Again, number of lines joining 2 points is:t2ca:66
But 12 sides are not diagonal.
Solving (i) and (ii) we get,
Total number of diagonals = 66-12 = 54
n: 12 and r: 3
2. (w) How many triangles can be formed by
l. (ix) Prove that, n*'C, a n*lCr-r = !+29 joining the angular points of a plane figure of 16
Solution: n*tC. 1 n*lCr-r sides? How many diagonals it has?
(n + l)! (n+l)! Solution: A polygon of 16 sides has 16 angular
r!(n+ I -r)!' (r-l)! (n+ I -r+l)! points.
(n+l)! From the 16 angular points, joining any 3 points a
r(r-l)! (n + I - r)! ' (r-l)! (n+ 2 - r) (n + I -r)! triangle can be formed.

_ (n+l)! 4.= I \ So number of triangles formed by selecting 3 points


- (r-l)! (n + l-r)! \r - n +2-r) out of 16 points is r6C3 : 560 ways
Again, number of lines joining 2 points is: l6Cz:
(n+l)!
- (r-l)! (n + 1-r)! ' n+2-r*r
120.

r(n + 2-r) But 16 sides are not diagonal.


Total number of diagonals: 120-16:104
_ (n+2)(n+l)[
r(r- 1)! (n+ 2-r) (n +I -r)! (v) Show that a polygon of n sides has n(n-S)
I
: r! !n(n 1?)!
, ., : n*2c,.
+ 2-r)l
diagonals. Show also that the number of
different triangles formed by joining its angular
2. (i) In BPL 8 team play in a group..Find the
number of games thai will if one team
points i,
I n (n-1)(n-2). tDB. Osl
f.ku nl"ce
Solution: A polygon of n sides has n angular points.
plays one match with each of thi othSrs in group
By joining any 2 points, total number of lines is nC;.
round?
But among these 'n' lines are sides of the polygon.
Solution: 2 teams can be selected out of 8 teams in
tC,
or 28 ways.
.'.Number of diagonals : oCz-n :ffi. -"
2. (ii) In how many ways 4 digits which are not _ n(n-l)
- _n2 -n-2n
z -n- 2
divisible by 3 out of the digits l,2r 3, + 5, 6, 7, 8,
9 can be selected? lcB. 171 . :+:|n(n-3)
Solution: Digits which are not divisible by 3 are l, Again, number of triangles formed by selecting 3
'; .
2,4,5,7, 8 i.e. total 6 digits, poltt out of 'n' points ,r :_;;r-F
6Ca
.'. 4 digits can be selected out of 6 digits in or
l5 ways. 6
Chapter-5: Permutations and Combinations 297
,,
(ri) Cinu, seven straight lines of tength 1,2,3, l,l Z. (viii) Find the number of points of intersection of
5, 61 7 cm respectively; show that the number of | 10 straight lines drawn in a plane no two of the
ways ,in which four can be chosen to form a I straight lines are parallel and no three are
quadrilateral is 32. I concurrent.

[SB. 12, 08, CtgB; 12, ffi, 06, DiB. 09, I


Solution: 2 straight lines make a point of
RB. 10,04, BB. 10,.rB. 0g, DB. 031 I intersection.

solution: GivenTlinesarr 1,2,3,4,5,6,7cm. I So2snaightlinescanbetakenoutof l0int0c4ways'

4linescanbeselectedoutof7intoor35ways.^l.Totalpointofintersections_,oC,:ffi
But the sum of lengths of any three sides of a
quadrilateral is greaterthan fourth side. I l0 9
-::45
x x g!
L *,,
In case of combinations (1, 2, 3,6), (1, 2, 3, 7),
(ix) There are 15 points in space no three of
(1,2, 4,7) a quadrilateral cannot b€ formed since
which are in the same straight lines and no four
the sum of smaller three sides is not greater than the
fourth.
I tn the sarne plane with the exception of 5 points
which are in the same plane. Hry many different
.'. Number of quadrilaterals = 35 - 3= 32 I planes can be formed by those points?
a (vii) Find the number of straight lines obtained
I Solution: A plane is made of 3 points.
by joining 12 different points in a plane, no three
I . The number of ways that 3 points out of 15 can
of which are collinear with the exception of 4 | be selected is
points which are collinear. Find also the number
oftrianglesformedbythem. I _,r^ _ 15!
- Cs:@:- _ 15 x 14 x 13 x 12! _,<<
I 6xl2l
Solution: I't part: A straight line can be obtained by 5 points are coplanar, they form
joining any two points.
I Again, since
sc3.
1

I plane instead of
r5cr 5c3
+I
ilIl::ll:TlTil,T::"frH:ff;ffi;,n;;
.r6aur,
we get
elrlvw f yvrrrlJ u w wvrrlrrvq,
I aC2.
line instead of
vJ JvururE rrrvrrr I
I
rota'I number orpranes =
=
= 455
-
- l0 + l:446
, nC,
, +I I
I Z. (x) There are n points in space no three of which
.'. Total number of lines - ,,tzCz- are in the same straight line and no four in the
:ffi.ffi*tl.i..uo.".1.:.Forrvha,,:,:u,:'nwill,*i,:o:1 ,- _:_-_,__ ^ ^. _

of straight lines be equal to the number of planes


: lZxll x l0! 24 + I I
ffi 2xl0! -*-2x2'r I obtained by joining these points?
Solution: For a straight line 2 points and for a plane
:66-6+l:61 I ^ . ,-
2nd part: A triangte can be formJ by joinins 3 I :,i"]i:: T:::T1t:i
Here total points is n.
points' ,. I .'. r\r,'u\ of straight lines formed by selecting
" Number 2
of 12 canbe selected in l2Cg ways
3 points out I
Again, since 4 points are colline.r, ttry *itt rot -nCz:@P
I Oointsoutofnis
form a triangle. If they were not collinear, they
Number of planes formed by joining 3 points out of
would formtriangles.
aC3
I

...Totalnumberoftriangles:,,C,_oC,l,is:,C3.:eP
:frm-3(4-3x
t'Lt
l2t ''r I
| - .
4t
3y the question, number of planes
.
: number of lines.

6x9l '6xl
=220-4=216
Ir
I

or,j(n-2)':lor,n-2-3.'.n=5
|
298 Chapter-5 : Permutations and Combinations

3. (i) In how many ways ean a eommittee be formed 3. (vi) In how many ways can a selection of three
of 9 genthren, d * .hdies from a
consisting consonants and two vowels be made from the
board.consisting of 12 genthuat, ard 8 lady letters of the word 'Motherland'?
directors? Solution: The word 'Motherland' consists of 10 letters
Solution: 9 gentlemen eill be selecfed q+t of 12 in all of which are different. It consists 3 vowels and 7
'2C, or 220 ways. consonants
3 tadies can be selectedout gf,,s,in'C, or 56 ways. 3C2
or 3 ways.
2 vowels out of 3 can be selected in
.'. Total ways :220 x 56 : 12320
3 consonants out of 7 canbe selected in'C, or 35 ways.
3. (il) A candidate is to answer 6 questlons out of 12
ways:35 x 3 : 105
.'.Total number of
questions. He is to select exactly 4 questions from
4. (D In an electric generator eompany, 5
the first 5 questions. In how many ways can he
mechanical engineers and 4 electric engineers
select the questions?
are acting. In how many ways a committee of 4
sorutio,,oq"l.'r;*r';t;#;3i:[];I;11i3;l'l members can be formed consisting of at least I
mechanical engineer and I electric cngineer?
(6-4) or 2 more questions,rrn out of
remaining (12-5) : 7 questions
,b-.,,re]e9!ed
lri 'e, : 2! ways. solution: committee can be #f # ii:
.'.Total ways of selection :5 x 21 :.105 following ways:
3. (iir) A man has 12 friends of whom 8 are Mechanical (5) Electricat (4)
relatives. In how many ways can he invite 7 (a) I 5

friends so as to include 5 relativeg? (b) 2 2

IBUET. 12-131 (c) 3 I


tC, *
Solution: Number of non-relative' ftiends =(12-8) : 4
oC,
(a) Number of combinations: =5x 4:20
5 relatives out of 8 can be invited in ts'oi 56 ways. (b) Number of combinations :tQrx4Cz: l0 x 6:60
(7-5) = 2 non-relative friends out'of 4 ean be invited (c) Nunrber of combinations :
tC, * o'C, = l0 x4 : 40
inaC2=6ways. .'. Total number of ways :20 * 60 + 40
: 120
Required number of ways'= 56x.5+€36 (iDA cricket team of 11 players is to be selected
3. (iv) A board is madc',ep'of,ordchairenan, two from two groups of 6 and 8 players. In how
vice-chairmen and 16 otke* msmbens. In how many ways can the selection be made on the
many ways can a eomt$Gtee,of Sitp:fiormed so as supposition that the group of six shall contribute
always to include the,.&eismfirl,,4*d one vice- no fewer than 4 players?
chairman? lsB. 11, cB. 05, 01, BUET. ll-121
Solution: Selection of soilx*itlee Ss as,f.sJlows: Solution: Formation of teams are as follows:: .'

Chairman (l) Vice chairuran (2} , ,,M€mhers (16)


I't group (6) 2"d group (8)
I I .. : ;: ^ 1, ,3
(a) 4 7
Number of ways - 'C, xzCr ^'uC, (b) s 6
: I x 2x56A ='1120' (c)6s
3. (v) In how many wayq caR a selg.qtlqp of three (a) Number of combinations : uco * tC,
consonants and two vow.E,,h made from the :15x8:120
letters of the word 'TOGAHITIIMS'?'
(b) Number of combinations :
ucr r. *Cu
Solution: The word 'I.OGAIITHMS'.consists of 7
consonants and 3 vowels. ";
:'::,.'['ut
(c) Number of combinations
3 consonants out of 7 car be relected'in'C, or 35 ways.
2 vowels out of 3 can be selected in
3Cz
or 3 wayt.
: 1 x 56:56
.'.'total number of ways - 35 x 3 : 105 .'. Total ,ru*'b...of ways : !20+ 168 + 56:3M
{,,
Chapter-S : Permutations and Combinations 299
4. (iii) A candidate is required to aRswer 6 Required selections are as follows:
questions out of 10 which are divided into two Even (4) odd (4)
groups each containing 5 quesfions, and he is not (a)13
permitted to attempt more than 4 fnom any (b)22
group. In how many different ways can he makes (c)31
up his choice? (a)Numberofcombinations-oC, x 4Cs :4x4: 16
Solution: Probable selectio*r of the pstions,are as (b) Number of conrbinations: 'C, x 4Cz: 6 x 6 : 36
follows: (c) Number of combinations - oC, x 4Cr : 4x 4: 16
.'. Total number of ways :
I't group (5) 2od group (5) :
16 + 36 + 16 68
(a) 4 '2 : :.: r.' ,
4. (vi) The executive committee of a club consists of
(b)3 3
11 members of which 8 are females and
(c) 2 4 ,.,, : .
13 males. One female member is the president of
(a) Number of combinations : 5C4x:Ce : 5I 10:50
the club. In how many ways can a
(b) Number of combinations = tC.#?+:"10.xl0=100
subcommittee of l1 persons be formed so as to
(c) Number of combinations = sgrxsCr: l0x5:50
include at least 4 ladies except the president?
.'.Total ways: 50 + 100 + 50:200 .
[CtgB. l7l
4. (w) In how many ways can ,a committee of 5 Solution: Committee of 1l members can be formed
persons be formed so as.to include at least one in the following ways:
lady from l0 persons including 4ladies? Female (7,Except
lMB. 06, 05, BB. 04, JB.1.02npU. 12-131
Solution: Committee can be formed in the
following ways:

'::'
(a) 1 4
'3 -':"r":
O) 2 .'. Total number of ways
(c)32:' :(l3c7x7co * ,,cu x 7cs + t3csxTcu + ,rco*,cr)
(d)4 I : 60,060 + 36,036 + 9,009 + 715 : 1,05,820
(a) Number of ways -nC,
x 5Cl :4
x'Is':60 4. (viD For an one-day match NEWZEALAND
(b) Number of ways - ocr,x 6Cs: 6 x 20 =120 cricket team consists of 7 batsmen, 6 bowlers and
(c) Number of ways - 4Cr
x 6Cz=
4 i 15:60 2 wicketkeepers. In how many ways a team
(cl) Number of ways - oco, 6C,:1 x 6:6 consisting of l l players may be formed so as to
.'. Total number of ways = 60 + l2O +60 + 6 5 bowlers
include and at least one wicket
:246 keeper? [DjB. l7l
:
4. (v) How many different combinations'can be Solution: Team can be formed in the following
formed of eight counters marked lr 2, 31 3, 4, 5, ways:

617,8 taking them four at a time there being at Batsmen (7) Bowler (6) Wicketkeeper(2)

least one odd and one eyen counter in each (s)ss I


combination? (b)45 2
Solution: Out of total 8'counters,'"4 'ars even .'. fotat number of ways
marked and 4 are odd rnarked. = 'c; x 6cs x 2cr a|ca x 6cs xzcz:462
300 Chapter-S : Permutations and Combinations

4. (viii) Of 14 players 5 "are bowlers and 2 are Solution: The group can travel as following ways:
wicket keepers. In how many ways a cricket I't vehicle 2nd vehicle
team consisting of 1l players may be formed so (a)72
as to include at least one wicket k€eper and three (b)63
bowlers? BB, 14; M8.20001 (c)s4
Solution: Formation of&m.is as follows: (a) Number of selections = eCt = 36
Bowler(S) wicket keeper (2) Glrcrs (7) (b) Number of selections: : 84
eCo

(a)3 I :l (c) Number of selections - 'C, : 126


(b)4 I 6 .'. Total number of ways : 36 + 84 + 126 :246

(c)s I 5 5. (ii) A group of 10 students has come from


(d)3 2 6,, SYLHET to BANDARBAN in a study tour. They
(e) 4 2 : , ,,5 have to travel in two vehicles, one of which will
(f)s 2 4. not hold more than seven and the
(a) Number of combinations= scs.xbr {C*toxzx l=20 other not more than four. In how many ways can
(b) Number of combinations = tC+. r*?C 6=5x2x7=70 the group travel? uB. l7l
t 7Cr=
(c) Number of combinations: Cr r2C, x 1 r 2, 2l=42 Solution: l0 students can travel as following ways:
sca
(d) Number of combinations= x2crxTcu*l 0 x I x7 =7 0 I't vehicle 2nd vehicle
(e) Ntnnber of combinations = 5Cor2cznl
Cs=5xlx21 =l 05 (a)73
(f) Number of combinations = tCr"2Crr?Co=1x1v35=35
(b)64
.'.Total ways :20+70+42+70+ 105 + 35= 342 (a) Number of selections - ''C, : l2A
4. (ix) In how many ways cen a committee of 5 (b) Number of selections - 'ocu - 210
persons be formed so as to include at least .'.Total number of ways = 120+210:330
3 men out of 7 men and d women?'[RUET: 19-201 5, (iii) A party of 7 persons is to travel in two
Solution: Committee can be formed in the vehicles, one of which will not hold more than
following ways: seyen and the other not more than four. In how
Men (7) Women (6) many ways can they travel if any one of the two
(i)32 vehicles won't be without passengers?
(ii) 4 I :.: lMB. l1; JB.05l
(iii) 0 :, Solution: The group can travel as following ways:
(i) Number of ways --'Crx 6cr=,35.x 15:525 I't vehicle 2"d vehicle
(ii) Number of ways :7Cqx 6Cr:35 x 6:210 (a) 6 I
(iii) Number of ways -'Crx 6Co :21 x I
=21
(b) 5 2

.'. Total number of ways : 525 + 2l{} + 2I= 756 (c)43


5. (i) A party of 9 persons is to trsvcl in two (d)34
vehicles, one of whieh wiII not hold more than (a) Number. of selections:7Ca:7
seven and the other not more than four. In how (b) Number of selections :7Cs:21

many ways can they travel?. (c) Number of selections :1Cn:35


lDjB.la;
DB. 14, 11, 06, 01, JB..l1r.O5r CB. l0; S8..14, 10, (d) Nomber of selections : 7Ce : 35

04, CtgB. 09, BB. 09, 05, RB. 07, 05, MB. 09,031 .'.Total numbei pf ways = 7 + 2l + 35 + 35 : 98
Chapter-5 : Permutations and Combinations 301

6. In how many ways can a selection of 5 persons Solution: Each of 8 questions has an alternative.
be made from 12 persons (i) so as always to Each question has 3 options for selection.
include, fwo particular persons (il) so as not to Taking it, taking its alternative or rejecting both.
include two particular persons? .'. Options for 8 questions is = 38 r

Solution: (i) Including2 particular pirronr always, But among these 38 options, I option is such that it
3 more persons can be selected out of remaining l0 excludes all the questions (with alternatives) which
loC3 is not acceptable.
persons in or 120 ways.
.'. Required number of ways = 38 - I
(ii) Excluding 2 particular persons always, 5
persons can be selected out of remaining 10 persons
8. (i) F'ind the number of words which can be
formed with three different consonants and two
in roC5 or 252 ways.
vowels out of 12 different consonants and 5
7. (i) There are 10 candidates for 3 vacancies, and
different vowels. lCtgB. Oel
an elector can vote for any number of Solution: 3 gonsonants out of 12 can be selected in
candidates not more than the number of
''C, ways and2 vowels out of 5 can be selected in
vacancies. In how many ways can an elector tC, ways.
vote? lDB.09l .'.3 consonants and 2 vowels can be selected in
Solution: The elector can vote for 1 or 2 or 3 tcr=2200 wa,s'
"Cr'
candidates out of 10.
Number of words formed by 5 letters is :5! : 120
.'.Total number of ways - 'oC, * loCz + 'oC, .'. Total number of words :2200 x 120 = 264000.
:10*45+120
8. (ii) In how many words will all the vowels be
:175
present if different words are formed from the
7. (ii) In an examination a minimum number is to letters of the word 'CAMBRIDGE' taking 5
be secured in each of 6 subjects for a pass. In letters at a time? lCB. 07, CtgB. 081
how many ways can a studeilt fail? Solution: The word 'CAMBRIDGE' consists of 3

Solution: A student may fail in I or 2 or 3 or 4 or 5 vowels and 6 consonants.


or 6 subjects. Taking all the vowels, 2 consonants out of 6 can be
6C2
.'. Total number of ways selected in or 15 ways.

- uC, 16C2",. 6Ct + uCo + uC, + uCu Number of words formed by 5 letters is = 5! = 120

=6*15+20+15+6+l:63 .'. Total number of words = 15x120:1800.

7. (iii) A man has 6 friends. In how many ways can 8. (iii) Find the number of words which can be
formed with two different consonants and one
he invite one or more of his friends? [CB. 031
vowel out of 7 different consonants and 3
Solution: For each friend there are two options:
different vowels, the vowels to lie between the
Inviting him or not inviting him.
consonants.
.'.Options for 6 friends is:2x2x2x2x2x2=26 Solution: 2 consonants out of 7 can be selected in
But among these 26 options, I oplion is such that it
'Cror2l ways.
excludes all the friends which is not acceptable. I vowel out of 3 can be selected in3C1or3 ways.
.'. Required number of ways = 26 - I = 63 .'.2 consonants and I vowel can be selected in
7. (iv) Show that the number of sll possible 2l x3:63 ways.
of one or more'qutstions from eight
selections Again keeping vowel in the middle, 2 consonants
given questions, each question having an can b'e arranged in 2 sides in2t or2ways.
alternative is 38- l. .'. Total numbEr of words = 63'x 2=126
302 Chapter-5 : Permutations and Combinations

9. . (i) Find the number of cumbinations of the letters 9. (iii) In how many ways can the letters of thr
of the word 'THESIS' trkff.4 at a.fime. word 'EXAMINATION' be arranged by taking
4 letters at a time?
[DB. 13, 08, RB. 13, CB. 11, 06, 0g,
.), Solution: The word 'EXAMINATION' consists ol
j' ' l2'BB' 06' o3l I I letters.
, .: ,,',,,,P:jP'
Solution: Tlie word.:THESISI,cansrsts of 6 letters Letters are: AA, II, NN, E, X, M, T, O
of which 2 are'S's. This uryrdrsreists 5 different Combinations and permutations of 4 letters are as

'' follows:
letters. ']:-:'I::'
i': Description Combin Permutation
Lettersare: T,H,E J"',SS ., ation
,,,.,i,i;fu.,
Combinations of 4 letters,are as plfolp: (a) 2 alike,2 alike C2 3crx#i.
,

Description Spmbisatbn tCl x


(b) 2 alike, 2 'C2 3C,
*7cr*fi.
(a) 2 alike,2 different l xaer=6 different
(b) All are different 'C+: 5 (c) All 4 are
oCn
x4l
.'.Total number of osmbinatiogt-6 + 5 : II
different
8C+
9. (ii) In how many ways can the letters of the word .'. Total nwnber of combinations:'Cn*'Cr*'Cr.'

'PROFESSOR' be arranfttl 'by'taking 4 letters :94


at a time? Required number of arrangements or permutations
[S8.09, CtgB. 011

Solution: The word 'PROFESS6il' .oorists of 9 -'c,"#.*, a 3cr x?cz"#. *,cax {r'

= l8+756+ 1680:2454
Letters are: O O, S S, R R; P,:E,F 9. (iv) How many different words can be formed
Combinationi and peqnutations of'4 l€tters are as
from the letters of the word 6ADDRESSEE'
taken 4ata time? [KUET: 07-081
follows:
Solution: The word 'ADDRESSEE' consists of 9
Description Combina Permutation letters.
tbn , Letters are: DD, SS, EEE, A, R.

(a) 2 alike,2 alike 'Cz '41 Combinations and permutations of 4 letters are as
'czx znt follows:
Description Combi Permutation
(b) 2 alike, 2 'c, i tel' ' tin1":c'*fi nation
different .41
i!.r :
(a) 3 alike, I different lxaCl I x'C1x
(c) All are Ct 31

different
(b) 2 ahke,2 ahke 'Cz tc2x#i.
... Total number of combinationg=,f,7 * rCq x'C7 * (c) 2 alike,2 different 'c1 x
oc2
lcr*4Crr.r!,
=48 ,CO
.,, ,;:, :.-' (d) All.4 are different x4l
Required number of arrangem-qlt! gr per4utations
:
.'. Total number of Words
a.l at..
-'crxTii * rcr x)c2 x T)ta x 4! : l.x oc, r{i 13cz-#.a 3cr *acrrfi*tco, +,.
:18+360+360 -738 : 16 * 18 +216 + 120:370
Chapter-S : Permutations and Combinations 303

9. (v) In how many ways can the letters of the 9. (vii) In how maay ways can the letters of the
word 'PARALLEL' be :Arranged by taking 4 word'England' be arranged by taking 5 letters
letters at a time? ,,ii , at a time? [CB. l7l
Soltrtion: The word 'PARJTLLE[,,1 ,6nsists of 8 Solution: The word 'England' consists of 7 letters of
letters. ' which 2 are'rf s and remaining are different.
Letters are: LLL, AA, P, R, E Leffers are: nn, E, g, l, a, d.
Arrangements of 4 lettors ff ,fhe rvord are as Combinations and permutations of 5 letters are as
follows: follows:
Description 'iPcnntitation Description Combination Permutation

(a) Three L, one


t1'"# = to
(a) 2 alike, I x'C3 tcr*1
different 3 different
(b) l'wo L, two A 4l (b) All 5 are
uc, 6C5
x 5!
M=b different
(c) Two L, two ocr*fi:n .'. Total number of ways = + 6cs x 5!
different fa, " i)
(d) Two A, two .Ll
oCr*T, :600 +720:1320
:72
different (viii) Find the number of words which can be
(e) All are different 'Coi+! =l2o formed with two consonants and one vo\uel from
Required number of arrangements or permutations the letters of the word rPermutations', the
: 16* 6+72+72+ 120:286 vowels to lie between the consonants.

9. (vi) How nnany different words can be formed IRUET. t2-13; DB. lsl
from the letters of the word 'ENGINEERING' Solution: The word 'Permutations' consists of 5
vowels and7 consonants of which 2't's.
taken 4 at a [RUET: 10-t ll
tiine?
Solution: The word 'ENGINEERING' consists of I vowel out of 5 can be selected for middle position
5C1
in or 5 ways.
I I letters.
Letters are: EEE, f{I.fN, GG, II, R
(a) 2 consonants are alike (both are t): 2 't' out of 2
can be selected in'Cnor I way.
Combinations and permutationt,gf 4 letters are as

follows: .'. Number of words : r r*.x 5 :5


Description Combi Permutation
,
(b) 2 consonants are differbnt: 2 consonants out of6
nation
canbe selected rr.uCnor 15 ways
(a) 3 alike, I different 'cr x "cr 2C, xtcr*$ .'. Number ofwords: l5 x 2l x 5: 150
So total number of words :5 * 150: 155
(b) 2 alike,2 alike "C, 4czx#i.
10. (i) In how many ways can 20 different things be
(c) Zalke,2 different tr"G 4C, x,4crxt aii,iaeO into 3 groups of 8, 7 and 5 things?
Solution: Number of groups taking 8 out of 20 is :2oCe
(d) All4 are different
,CI
. tcox4! Number of groups taking 7 more out of 12 is : r2Ct
.'. Total number of words Number of groups aking last 5 things is : sCs
.r,
ac3 o"rrL * ac1*acr*#'*sco x 4!
.:. Total number of groups :"r?rr *'lr,,r*rt
-'crx x
*.* 2ot
:32+36+288 +120:476 - 8!12! '7t5t '' r I-
8!7!5!
304 Chapter-S: Permutations and Combinations
10. (ii) In how many ways can.l8 different things be ll. Find the number of ways in which p positive
divided into 3 groups so thatsch group has 6
things? arranged in a line so that no two positive signs
Soluti6n: l8 different thingq,w{ff,be,divided into 3
shall be together. l
groups where each group has-,6 thingS,
Solution: q negative signs can be arranged in a row
18 things can be divi&d into 3 Eroups i, # in I way.
ways. But 3 groups can be manged among In their middle (q l) positions and at the
themselves in 3! ways. beginning and at the end 2 positions i.e. in q- I + 2
: q + I positions p positive signs can be arranged in
.'.Required number of ways: t*$ (q + l)!
10. (iii) In how many ways can 15 prizes be divided --'up:m;GF
: - q+rr.
ways'

12. Three flags are required to make signal. How


Solution: If
prizes are dividiO iirto 3 groups then
15
many signals can be made with 24 flags of six
each group will contain 5 prizes.
different colors each four flags being of the same
l5 prizes can be divided into- 3 grffips to color?
ffi*uVr.
But 3 groups can be arranged among themselves in Solution: The signal can be made in following ways:
3! ways. ,r,:.: i (D All 3 are of same color.
(ii) All 3 are of different color.
.'. Required number of ways'= Jeit
(5!)'3! (iii) 2 are of same color and I is of different color.
10. (iv) In how many ways can 3 committees of 5
members each be formed from a total of 15 .'. Total number ofways : 6x I +6Ps * 6 x'C, *#
students? [CUET.05-061 : 6 + 120 +6 x 5 x 3 : 126 +90 :216
Solution: 15 people can be divided into 3 groups
.(ls)! 13. In how many ways can a boat's crew of 8 men be
in
ffi ways.
:

arranged if 3 of them can only row on one side


But 3 groups can be arranged among themselves in and 2 on the other side?
3! waYs' Solution: Out of 8 crews 3 can row on one side and
,'. Total number of ways :
lii ,: ,

2 can row on other side.


(rft
.'. Remaining 3 can row on both sides.
10. (v) In how many ways can 52 ca1ds be divided
equally among 4 persons? .'. 3 or 4 or 5 or 6 crews can be arranged in first
Solution: Each of 52 cards are dft&rent. If these side and 5 or 4 or 3 or 2 can be arranged in second
cards are divided among 4 pelsons equally then side.
each will get 52 + 4: 13 cards. , . Again 2 crews can row in both sides in 2l or 2
Itt person can be given in 'C,, ways.
ways.
From remaining (52-13):39 cards,
.'. Total number of arrangements
2'd person can be given in 3eC13'wfiys.'
From remaining (39-13)= 26 cardq' ; : i'' I
:r(3! x5! +4! x4! +5! x3!+6tx2t)
3'd person can be given in,=,26Gr*,w&f,gii,1, I

From last (26.13) 13 cards, : :: r,,, : , :, 1

(6 x 120 + 24 x24 + 120 x 6 * 720 x 2)


4tr person can be given
in
13C13 :
wals. ,:, .,
.'. Total number of ways .'*
".:" .l:i
: 52C 13C13 Q20 + 576 + 720 + 1440)
r lx3eC 13x26C 13x
szt 39t 26t
=J3m
'szl I
=, * 3456:-1728
ffi'l3tt3t 'r=Gi)a
Chapter-S : Permutations and Combinations 305

Creative Creative Ouestion-2


Recently DG instructed evefy mllege to update The executive committee of a club consists of I I
PDS file. Accordingly our cotrlcagte Mr; Khan members of which E are females and 13 males.
has used his user ID "COFBtr{ATIOI{'' and [CtgB. t7l
passwod'610652'. , IDB 171 (a) Find the number of different arrangements
(a) If ? = 5{ and oC, = 9 thcn find'therehre of r. of the lettens of the word 'DEPRESSION' where
(b) How many differcnt arrdrgoments ,can be all the vowels wil'l remain together.
made from the letters of 'user IIP talcn 4 at a (b) One female member is the president of the
time? club. In how many ways can a sub-committee of
(c) How many odd numberc of five cfnilicant l1 persons be formed so as to include at least 4
digits can be formed with the dfiits of plssword, tadies except the president?
using each digit only once ln a nirmber? (c) Excluding'2 male members, in how many
wiys the remaining nembers can be arranged in
sorution: E'*. : 54 =6ff$ 54 ............ (D
a line so that two female members will not
occupy consecutive positions?
Sotution: fne word'DEPRESSION' consists of
I
(i)+(ii)+r! =6 :+r! =3! .'. r=3 l0letters of which 4 are vowels (E, E, I, O). ' :

E *r word 'COMBINATION' consists of l l The word DEPRESSION' consists of l0 leffers of


letters which are as follows: which 4 are vowels (E, E, I, O).
OONNIICMBAT Considering 4 vowels as I letter, total nurnber of
Combinations and permutations of 4 letters are as
follows:
letters is 7 and the number of arrangement isJf
Description Combination Permutation Again the vowels can be ananged among
(a) 2 alike, 'C, ,crr# themselves in
LI
11ways.
2 alike
o) 2 alike, 'C1'x'C2 3crx7qrfi .'. Total number of arrangement uftere the vowels
2 different
(c) All4 are
oCo t6f x 4!
occur together is =
;l *{r:30240
different I Co..ittee of I I members can be formed in the
.'. Total number of arrangement following ways:
3c2x,V#i.' 3cr xTczr*, *tca x 4! Male (13) Female (7,Except president)
(a) 4 t3C7*'co
7
= l8 * 756 + 1680 = 2454 (b) 6 5
l3c6r'c5

lf, civen digis: l,0,6, 5,2 (c) 5 6 "c5*'c6


Odd numbers have odd digits in unit place. So (d) 4 7
ttCa*'C1
required numbers will have I or 5 in the unit place.
.'. Total number of ways
Keeping 5 in the unit place, number of permutations : (l3czx7co* "cu x 7cs + "cs*'cu + l3cox7c7y
with remaining 4 digis is : 4! :24.
= 60,060 + 36,036 + 9,009 + 715: 1,05,820
Again numbers starting with 0 will not be of 5
digits. ! fxctuding 2 male members rernaining male
.'.Keeping 5 in the unit place and 0 at the fnsL members is (13- 2) = l l
I number of permutations with remaining 3 digits is I I
male members will be arranged in a line so that
4
g 3! :6. .'-- tyvo female members will not occupy consecutive
,l .'. 5 digit numbers =24-6=18 positions.
v
4 Similarly, keeping I in the unit place 5 digit In this line of I I
male there are l0 positions
GI
numbers = 18 between them. At the beginning and at the end there
$
.; : : are2 more positions.
.'. Total 5 digit odd numbers 18 + l8 36

it
306 Cirapter-S : Permutations and Combinations

lhese
These (10 + 2) : 12 positions ;an be filled by 8
r2Ps
Creative
bmales in
femal ways.
For an one-day match NEWZEALAND cricket
\gain, I I males can be arranged in that line in I I !
Again
team consists of 7 batsmen, 6 bowlers and 2
ways.
vays.
wicketkeepers. lDjB. r7l
'. Required number of arrangements is: l2Ps
x II! (a) ff nP, = 2 x"Cothen find the value of n.

ffi (b) In how many ways a team consisting of 11


players may be formed so as to include 5 bowlers
)hysicist Issac Newton born in England
Physi in 1643. and at least one wicket keeper?
lcB. r7l (c) Find the number of arrangements that can be
a) I'In how many ways 4 digits which are not
(a) made of the letters of the word
livisible by 3 out of the digits 1,2,3, 4,5, 6,7,8,
divisi 'NEWZEALAND' so that the vowels may not be
9) can be selected? in consecutive positions in any of them.
(b)
b) sl
Show that, the permutations of the letters of Solution:[ Given, 'P, : 2 x"Ca
the last part of the name of the seientist taken all
ot, n!-n!:2x
he la
atat
rt a time is 12 times that of first part. 4l (,-4r
(c) In how many Ways caqthe fe{ters of the birth
c) In
llace of the scientist in the scenario be arranged
place or,#:2x#f
by taking 5 letters at a time?
ry tal
1t
iolution:IOigitr which are not divisible by 3 are
Solut
t
or,
ffi: lZ or, n- 3 = 12 .'. n: 15
,2,4,5,7,8
1.2. i.e. total 6 digits.
'. 4 digits can be selected out of 6 digits in 6Ca or p f.urn can be formed in the following ways:

15r
5 ways. Batsmen (7) Bowler (6) Wicketkeeper(2)

trE ,u word 'Issac' consists of 5 letters of which 2 (a)ssl


are
|re s. (b)452
'. Number of permutations i, = : oo .'. Total number of ways
ff r 6cs x 2c,
47ct * 6cs x2cz:462
,ast word'Newton'consists of 6 letters.
Last
-'cr
'. Number of permutations is : 6! :720: 12 x 60 I ffr. word 'NEWZEALAND' consists of l0
: 12 x permutation of lst word letters of which 2 are'E' s, 2 are 'A' s, 2 are 'N' s and
'. Number of permutations of the last word is 12 remaining are different.
times t of first word.
imes that
Here, N and n are considered as different letter .'. Total number of permutations :*,r^:453600
IHere,
ince oone is capital letter and other is small letterl
slnce This word consists of;4 vowels which are E, E, A, A.

I ffr.
Ern word 'England'consists of 7 letters of which 2 Considering 4 vowels as I letter, total number'of
re
are h's and remaining are different. letters is 7 and the number of arrangement is
,etters are: nn, E, g, l, a, d.
Letters 11
lombinations and permutations of
Combi 5 letters are as
:fi.:2520
followr
bllows:
Again the vowels can be arranged among
Description Combination Permutation
A]
(a) 2alike, I xrC, themselve s in gr: 6 ways.

3 different
tcr r#
.'. Totai number of arrangement where the vowels
(b) All 5 are "C 5 6c, *
5! : :
different occur together is 2520 x 6 l5 120

{c, x 5! .'- The number of arrangements where the vowels


. Total number of ways: (", ""r) +
donot occur in consecutive positions is
:600 +720= 1320 :453600
- 15120:438480
Chapter-5 : Permutations and Combinations 307

Creative Creative Ouestion-6


Scenario-l: Sadat can write the digits 0,3,4,5,6,9.
'BANGLADESH' is our motherland.
[All Board. l8l
.',\ ,\+,\AA (a) If nPl = 210 then find the value of n.
Scenario-22 A= 3i + 2j + 6k and B: i - 4j - 3k
nP, nC3 (b) Find the number of arrangements that can be
(a) If =3x then find the value of n.
made of the letters of the quoted word so that the
(b) Find the component of il abng i .
(c) From scenario-l, using each digit only once in vowels may not be in consecutive positions in any
each number how many signilicant even numbers of thdm.
containing six digits?
(c) How many different words can be formed
Sotution:flnPr:3xnC3
from the letters of the quoted word taken 4 at a
*J.!--2 i x
n!
= 2,. (" -2)l: 3l(n L)l time?
_ _f_:. I
Solution: E "r, :210
=ffii:jx3"r(n-3I
ll
+(rLXn-rt :m = n(n - l) (n - 2):210
I
= (n' - n) (n - 2):2lO
+n1:l=n-2-1.'.n:3 = n3 -2n2 - n',+ 2n:210
Ei = 3? + zj + ot, il = i -+i - rt =)n'-3n2+2n-2lo:o
+ n' -7n2 + 4n2 - 28n + 3on - 2lo: o
-r+'+-+
Component of6 along I i, : B coso ;:t* a
+ n'1n - 7) +4n(n - 7) + 3o(n - 7) : o
NowA:\/tT4=G:114s:7 +(n -7)(n' +4n+30):0
e:r,lil:iZ:iD :\m
.'.n-7:0or n'+4n+30:0
i .d : 3 x l +z x(-4)+6(-3)-3 - 8 - l8 = -23
.'. Il: 7 or n: (not acceptable)
.'. Required ;:+*
component:+
n:
23 lt*z?+ot_ n A A
Ans: 7
:-7= 7 --4r(3i+2i+6k)
n
fne word 'BANGLADESH' consists of l0
I
! Ciu., digits: o, 3, 4, 5, 6, g letters of which 2 oA's and all others are different.
Even numbers have even digits in unit place" So
.'. Total number of permutations : 1814400
required numbers will have 6 or 4 or 0 in the unit #:
place.
Again, this word consists of 3 vowels.
Keeping 6 in the unit place, number of permutationi,
with remaining 5 digits is 5! = 120. : Considering 3 vowels as I letter, number of letters
Again numbers starting with 0 will not be of 6
is 8 and number of permutations is: 8! = 40320
digits.
.'. Keeping 6 in the unit place and 0 at the first, But 3 vowels can be ananged among themselves in
number of permutations with remaining 4 digits is 3!
J ways.
4t:24. u.rr,
:120-24:96
.'. 6 digit numbers .'. Number of permutations keeping vowels together
Similarly, keeping 4 in the unit place 6 digit
numbers:96 is:40320x3:120960
Again, keeping 0 in the unit place 6 digit numbers .'. Number of permutations that the vowels are not
= 5l: 120 :
.'. Total even numbers : 96 + 96 + 120:312 together is lgf+OOO - 120960: 1693440
=.tl-

308 Chapler-S : Permutations and Combinations

E *r word 'BANGLADE-€I{' consists of l0


ll *. word 'AMERICA' consists of 7 leffers of
letters of which 2 'A's and a[ olhers are different. which 2 arc 'A's and remaining are different.
Irttbrs are: AA B N G t O E S H Lettersare:AAMERIC
Combination and 'df 4 letters of this Combination and permutation of 3 letters of this
word are as follows: :: - | word are as follows:
Description Combination Permutation
(a) 2 alike, I x'C1 x 5Cr x#
I
I different
oC3
(b) Alt 3 x 3!
are
different
.'. Total number of permutations
... Totat number of words : 1 ,,,i& r.,r{r*eco * 4!
: I x'Cr, ,!r*u", x 3! = 15 + 120: 135
:3?6 + 3924: 3360
Creative
gAMERICA' The word 'examinationt consists of 1l letters.
is a.powerful cryneqof the world.
(a) If 'C2 = l0 then lind the value of n. (a) If 'Cs = nCl then find the value of oP6.

@) Find the number of *ays"in which the letters


(b) Find the number of arrangements thlt can be

of the quoted word may be rearranged without made of the letters of the quoted word so that the
changing the positions of eny vowel? consonants may not be in consecutive positions
(c) In how many ways,cen the letters of the in any of them.
quoted word be arranged by taking 3 letters at (c) Find the number of combinations of the letters
a time? of the quoted word taken 5 at a time.
iorri"",E ncr==
lo Solution:[ Given nCs = nCr

1 il -rv
-!(n--l)_,.. SOn:8+3:ll
.'. nPu - t'Pu = 332640

=4#: lo =r n(n - l) =20 ' i;''' E *. word 'Examination' consists of I I letters of


" which 2 are 'n's,2 'i's,2 'a's and remainin! all are
+n'-n- 20:0
different.
+n2-5n+4n-20=,0
=+ n(n - 5) + 4(n - 5):0 .'.Total number of permutations:
#.:4989600
+(n-5)(n+4)=g . This word consists of 5 consonants.
.'. n-5:0orn+4=0 Considering 5 consonants as I letter, number of
.'. n=5 orn --4(notacceptable) 1l
n:5
Ans:
letters is 7 and can be arranged h r*-= 12,60 ways.

E *r word 'AMERICA' cqnpists .of 7 letters of Again, the consonants can be arranged among
which 4 are vowels and 3 consonants. . st
Since vowels will not drangethhpsition, they
themselver ir;i or 60 ways.
are
out of consideration Number of arrangements that consonants will
So 3 consonants can be arranged at9ong themselves remain together is = 1260 x 60 = 75600 a

in 3! or 6 ways. .'. Numberof arangements that consonants will not:


.'.Number of rearrangement is':6 - I = 5 remain together is :4989600 - 75600:4914000
Chapter-S : Permutations and Combinations 309

word 'Examination' consists of I I letters of Ttre word 'PERMUTATION' consists of II


lThe !l
which 2 are 'n's, 2 'i's, 2 'a's and remaining all are letters of which 2 are 'T's.
different. RM U A I O N
Letters are: TT P E
Lettefs are: aa ii nn E x m t o Combination and permutation of 3 letters of this
Combination of 5 letters of this word are as follows: wgrd are as follows:
Description Combination Description Combinati Permutation
uC1 on
(a) 2 ahke,2 alike, I different 'C2 x
o) 2 alike,3 different 'c1 x 'c3
(a) 2 alike, I I xeCr
I xeCl
3!

(c) All 5 are different 'C,


different "il
.'. Total number of combination is
(b) All 3 are
IUA
U3 ''c3 x 3!
different
-3Czx
6Cr
..3Cr xTCs+ tC,

#.
roC3
= 179 .'. Number of words: I x'C, , x 3!

ffi :27 *720:747


The word fPERMUTATION' means arrangement Creative Ouestion-l0
(a) The mobile numbers of GP fornted with l1 5. The number of teachers in 'Mathematics' and
digits of which first three are code number 017. 6Physics' departments of Adamjee Cantonment
How many mobite connections can be given with
College are 6 and 5 respectively.
this condition?
(a) Find the number of arrangcments of the
(b) In how many ways the letters of the quoted
letters of the znd quoted word taken
word be arranged so that the vowels may occupy
all at a time.
even positions?
(b) In how many ways can the selection be made
(c) How many.different words can be formed
from the letters of the quoted word taken 3 at a to form a committee of 8 teachers so as to give a

time? majority of Mathematics teachers in all cases?

Solution: frfoUile numbers are of I I digits and (c) Find the number of combinations of the letters
I
first 3 digits are fixed. of the l't quoted word taken 6 at a time.
.'. Number of changeable digits is (l I - 3) or 8. Solution: ![ ff,t word 'PHYSICS' consists of 7
Again, total mathematical digits is l0 letters of which 2 are 'S's and remaining are

.'. Number of ways of connection is 108. different.

I ffr. word 'PERMUTATION' consists of I I


.'. Total number of permutations ,r=*r= 2520
letters of which 5 are vowels and 6 consonants.
The positions that required to fill are shown below: I rn. committee can be formed in tlre foilowing
o@@@@@@@@@ @ way:
5 odd positions can be filled by 5 vowels in 5! ways. MATH (6) PHYSTCS (s)
There are2'T's out of 6 consonants. (a) 5 3

.'.Remaining 6 positions can be filled by 6 (b) 6 2


.6t
consonants rnil ways. (a) Number of ways =
oC5
x 'C, = 69
"(b) Number of ways: uCu
* scrl: l0
.'. Total number of ways : 5! x &ZOO
ff= .'. Tstal number of ways:60 + l0 = ?0
310 Chapter-S : Permutations and Combinations

I The word 'MATHEMATICS' consists of I I (c)Number of ways : 'Cr*'C,*7cr:1*2r2142


leffers of which 2 are 'M's, 2 'T's, 2'A's and (d) Number of ways : tcrr'CrxTcu=16*1x7:70
remain ing are different. (e) Number of ways : tcor'Cr*7cr:5*lx2l:105
Letters are: MM TT AA H E I C S (Q Number of ways : tcr*'Cr*7cn:1"1r35:35
Combination of 6 letters from this word are as .'. Total nunrber ofways : 20 + 70 + 42 + 70 + I05 + 35
follows: :342
Description Combination ! ffr. word 'BANGLADESH' consists of l0
(a) 2 alike,2 alike,2 alike ,C, leffers of which 3 are vowels andT consonants.

(b) oC2 Among3vowels2we'A's.


2 alike, 2 alike,2 different 'C2 x
Considering 3 vowels as I letter and 7 consonants
(c) 2 alike,4 different ,C, x ,Co
as another I leffer, these 2 letters can be arranged in
ocu 2, or 2 ways.
(d) All 6 are different
' rotar number:l:":T:'1":.;13cr Again, vowels can be arranged among themselves
.3!
m,r or J ways.
xTct 18cs
:179 And consonants can be arranged among themselves

ffi in 7! or 5040 ways.


.'. Totalnumberof ways:2x3 x 5040:30240
14 members 'BANGLADESH' cricket team Creative Ouestion-l2
consists of 5 bowlers and 2 wicketkeepers.
(a) How many numbers lying between 4000 and
The digits used in Mathematics are l, 2r3,4,5,6,
7181910
5000 can be formed with the digits lr 2, 3, 4, s
(a) Find the number of arrangemenh of the letters
with repetition of digit? of the word 6Cricketer' taken all at a time.
(b) In how many ways a cricket toam consisting (b) If the first five digits represent the length of
of 11 players.may be formed so as to include at five straight lines then show that the number of
Ieast one wicket keepei and three bowlers? ways in which three can be chosen to form a
(c) In how many ways can the letter of the triangle is 3.
quoted word be arranged so that the vowels and (c) Find the number of numbers less than 1000
consonants will be separately together? and divisible by 5 which can be formed with the
Solution: digits of the stem where each digit occurring not
I Nu*bers between 4000 and 5000
more than once in each number.
start with 4.
So remaining 3 positions can be filled by 5 digits Solution: I ffr. word 'Cricketer' consists of 9

with repetition in 53 ways. letters of which 2 are 'C's, 2 'e's, 2 'r's and

.'. Total numbers is:53 : 125


remaining are different.

team can be formed as follows:'' .'. Total number of arrangements is :#.:45360


ICri.t.t
bowler(S) wicket keeper(2) others (7) fen$h of 5 straight lines are l, 2,3,4,5
(a)317 I units.
To form triangle, 3 lines can be selected out of 5 in
(b)4t6
(b tC, or l0 ways.
(c)515
(c But the sum of any two sides of a triangle is greater
(d)326
(d than the tirirO.
(e)425
(e , .'. In theT combinations (l ,2,3),(1,2,4), (l ,2,5),
(0524
(0
?, 4), (1, 3, 5), (1,4,5), (2,3, 5) triangle cannot
(1,
(a)
(a. Number of ways :'Cr*'C,r7Cr:16*2xl:20 be formed
(b) Number of ways
(b :t|:o*'Cr*?Cu:5r2x7:70 .'. Numbe. oi*uyt : l0 - 7 :3 ''
Chapter-5 : Permutations and Combinations 3ll
Required numbers will have 0 or 5 in the unit t-ast seven digits: 1,2,3, 4, 5, 6, 8
I or 2 or 3 digits.
ITo form quadrilateral 4 lines of 7 can be
place. Numbers can be of out
7Ca
(i) There is I number of I digit which is divisible selected in or 35 ways.
bv 5. But the sum of lengths of any three sides of a
(ii) Keeping 5 in the unit place, I more digit for the quadrilateral is greater than the fourth.
tenth place out of remaining 9 digits can be So in the 5 permutations (1, 2 3, 6), (1, 2,3, 8),
ananged in
eP, :9 ways. (1,2,4, 8), (1,2,5, 8), (1, 3, 4, 8) quadrilateral
But I number starts with 0. cannot be formed.

.'. Keeping 5 in the unit place total2 digit numbers .'.Number of ways:35 - 5:30
:9 -l:8 I Civen digits: O, 1,2,3,4,5,6,8
Again, keeping 0 in the unit place, I more'dtgit for numbers have even digits in unit place. So
-Even
the tenth place out of remaining 9 digits can be required numbers will have 8 or 6 or 4 or 2 or 0 in
arranged in
eP1 :9 ways. the unit place.
Keeping 8 in the unit place, number of permutations
.'.2 digits numbers that are divisible 5 is in total
with remaining 7 digits is 7!
=8+9=17
(iii) Again numbers starting with 0 will not be of 8
Keeping 5 in the unit place, 2 more digits out
digits.
:
of remaining 9 digits can be arranged in ePz: 72
.'.Keeping 8 in the unit place and 0 at the first,
ways.
number of permutations with remaining 6 digits is 6!
But starting with 0 and keeping 5 in the unit place :
number of arrangements is:8Pr : 8
.'. 8 digit numbers - 6!) or 4320
(71
Similarly, keeping 6 in the unit place 8 digit
.'. Keeping 5 in the unit place total 3 digit numbers :4320
numbers
=72-8:64 Keeping 4 in the unit place 8 digit numbers 4320 :
Again, keeping 0 in the unit place, 2 more digits out Keeping 2 in the unit place 8 digit numbers 4320 :
of remaining 9 digits can be arranged in eP2 72 : And keeping 0 in the unit place 8 digit numbers: 7!
ways. or 5040.
.'.3 digits number! that are.divisible 5 is in total .'. Total numbers : 4320 + 4320 + 4320 + 4320
:64 + 72: 136 + 5040 :22320
.'. Total required numbers : (l + 17 + 136): 154
Creative Ouestion-1,4
Creative Ouestion-l3 Two cars are hired from 'BROTHERS'
0rlr2r3r 41 51 6r g MOTORS COMPANY, one of which will not
(a) If 4 x"C2 - nP, then find the value of n. hold more than eight and the other not more
(b) The last seven digits represent the lengths of than four.
seven straight lines. Find the number of ways in (a) In how many ways can 8 different pearls be
which four can be chosen to form a strung on a band so as to form a necklace?
quadrilateral. (b) If a party of 9 persons is to travel in the two
(c) How many even numbers of 8 signiflcant digits cars, then in how many ways can they travel?
(c) In how many ways the letters of the quoted
can be formed with the dtgits of the stem using-
word be arranged so that the vowels may occupy
each digit only once in a number?
even positions?
:'P3
!+ x 'C:
Solution:
Solution:[. Fixing I pearl, remaining 7 pearls can
_.,..n(n-l)_ be arranged in 7! ways.
=4xT:n(n-l)(n-2) Again for clockwise and for anti-clockwise
:+2n(n- 1)=n(n- 1)(n-2) .
permutations, same necklace is formed.
)2:n-2; [n*0,n* l] :+z :
.'. Number of ways 2520
... n:4
312 Chapter-S : Permutations and Combinations

The team can travel as fol lows: Putting r:2 in (i) we get,
I"'car 2"o car
(a) 8 I 6$1:182=W=r82 :

(b) 7
,, :+n(n-l):182 +n2-n-182=0
:0
(c) 6 3 = n'- l4n + l3n - 182

(d) 5 4
:+ n(n - 14) + l3(n - 14;: O

+(n-14)(n+13):0
.'. Total number of ways - 'C, .'- eCz .t eCo.t eCs ...n_14=0orn*13:0
=255 .'. n: 14 or n: -13 (not acceptable) 'l

Etnu word 'BROTHERS' consists of 8 letters of ...n+r=14+2:16


which 6 are consonants and 2 vowels.
Among 6 consonants 2 are 'R's.
! fn. word 'PARALLEL' consists of 8 letters o1
which 3 are 'L's and 2 'A's.
The positions that required to fill are shown below:
Letters are: LLL AA P R E
o@o@@@o@ Combination and permutation of 5 letters of this
aP2
4 even positios can be filled by 2 vowels in
word are as follows:
ways.
Description Combination Permutation
Remaining 6 positions can be filled by 6 consonants
(a) 3 alike, lxl r*l*3fr
.6!
m, ways. 2 alike
(b) 2 alike, lxlxrCl lxlx'C1
.'. Total number of ways - nP,
*ff: $20 2 alike, 5!
7nt
I different
Creative Ouestion-l5 zCr x
(c) 2 alike, "Cr 2cr x 4c3 x;i
(i) 'P" = 182, nC,
= 91 3 different
(ii) The word 6PARALLEL' consists of 8 letters. (d) 3 alike, 1 1{Cr nC,
t,
(a) How mirny passryords can be made up with 4 2 different
"ii
digits? (e) All 5 are
tCs
5x5!
(b) F'rom (i), find the value of n * r. different
(c) How many different words can be formed
.'. Total number of permutations
from the letters of the quoted word taken 5 at a
: I x l r# * I x I x3c1 q2cr"'cr";i
time?
Sotution:![ Total mathematical digits = 10
"h
Password of 4 digits can be formed in lOa or *tx ncr*lf *'c,"s
10000 ways.
= l0+90+480 +120+120 =820
n!
, .t = 182.......... (i)
GivennPr= 182 .'.'(n-r)! Creative Ouestion-16

.-.-L:61 0rlr2131415r7 are some mathematical digits.


Again, nCr:91 "r!(n-r)! 't ............... (iD
(a) Find the number of arrangements of the lettel
Dividing (ii) by (i) we get, of the word'Kolkata'.
n! (b) How many even numbers of 6 signilicant digil
(n-r)! :91
:W, _ n! ..(n-r)! can be formed with the digits of the stem usin
r!
_-d_ +ii Gi-rI
*J! - I
2 each digit only once in a number?
(n - r)! (c) If the digits except 0 represent the lengths t
ll -different straight lines then find the numbei t
=i:, alt:)
ways in. which four can be chosen to form
:+r! :2! :.r=2 quadrilateral.
Chapter-5 : Permutations and Combinations 313

Solution:I fne word 'Kolkata'co-nsists of 7 tetten 150


ofwhich 2 are 'k's, 2'a's andremaining are different. 75
25
.'. Total number of permutations =#. : 1260
5

I fr.n numbers have even digits in unit place. So Here among prime factor 5 is repeated 2 times.
required numbers will have 4 or 2 or 0 in the unit :.p=2
place.
Again, different prime factors are2 and 3.
Keeping 4 in the unit place, number of permutations
:. k=2
with 5 digits out of remaining 6 digits is:6Ps
Total factors of 150 is = 2k (p + I ) - I
Again numbers starting with 0 will not be of 6
=22(2+ l)- I
digits. : ll
.'. 4 in the unit place and 0 at the first,
Keeping
finat team can be formed as follows:
number of permutations with 4 digits out of I
All.
bowlerrounoer
remaining 5 digits is
5Pa Batsman wicket
So keeping 4 in the unit place 6 digit numbers
(8) (5) (4) keeper (3)

:6Ps-tPn=6oo (a)7sr2
Again, keeping 4 in the unit place 6 digit numbers (b)7422
:600 (c)7332
And keeping 0 in the unit place 6 digit numbers (d)7242
- 6Ps = 720 (a) Number of ways - tC, x sCs x 4Cr x3Cz
.'. Total numbers : 600 + 600 + 720: 1920 :96
lOigitt 1,2,3,4,5 andT
except 0 are tC, x sCo x4Czx 3Cz
(b) Number of ways -
To form a quadrilateral, 4 lines out of 6 can be :720
6Ca pr l5 ways.
selected in tC, x 5Cr x 4C: x3Cz
But the length of any thrde sides of a quadrilateral is
(c) Number of ways -
:960
greater than the fourth.
(d) Number of ways = 8Cz xsCzx 4C+
x3Cz
.'.In the combinations (1, 2,3,7) and (1, 2, 4,7)
= 240
quadrilateral cannot be formed.
.'. Total : 96 + 720 + 960+ 240 : 2016ways
.'.Number of quadrilaterals =15 - 2: 13
Ifnr word 'Bangladesh' consists of l0 letters of
Creative Ouestion-I7 which 7 are consonants and 3 vowels. Among
vowels 2 ate'a's.
20 members (Bangladesh' cricket team is formed
.'. Consonants can be ananged among themselves in
for 2017 Champions trophy of which 8 are
batsmen, 5 are bowlers, 4 are all-rounder and 3 7t or 5040 ways.
are wicketkeepers. Again, vowels can be arranged among themselves
(a) Find the total number of factors of 150. .3!
,nTorJways.
(b) In how many ways the final team consisting
.'. Total number of ways:5040 x 3 : 15120
of 15 players may be formed so as to include 2
wicket keeper and 7 batsmen? Creative Ouestion-l8
(c) Find the number of ways: in which the letters 2,31 41 5, 6 are some mathematical digits.
of the quoted word may be arranged (a) How many arrangements can be made out of
, without changing the relative positions of the the'letters of the word 'Director' all. of which
vowels and the consonants? have 5D' at the beginning?
3t4 Chapter-S : Permutations and Combinations

6) ffre digits lerffi,effive'straight


represent the Solution:'EOut of4, one candi&ietwill be elmted.
lines. Find the numbe_1. of ways in which three One candidate ean get utmost 7 votes.
can be chosen to form a triangle. .'. Number of ways 47 : 16384.
(c) How many numbers of 3 digits can be formed I ffr.word 'Engineering' consists of I I letters of
with the digits of tte rs*Gm rsfug,'a,@it any which 3 are''e's, 3 'n's, 2.'i's and 2'g's. .r*=
llt
number of times and which consist of two or .'. Total number of permutations : :277200
more of the same digits?' , :, ,' l
l,ifft
This word consists of 5 vowels and 6 consonants.
Solution:
E'O' will be at be beginning, so it will
Considering 6 consonants as I letter, number of
beoutofconsideratiol..,,, . :. .:.
6l
Accept 'D', there are 7 more letters out of which 2 letters is 6 and can be arranged inffior 60 ways.
axe'r's.
*i,
Again 6 consonants can be arranged among
1l
.'. Number of permutations : 252A ;ts i. #
i.= themselues or 60 ways.

5 straight lines are of lengtii 2,3,4,5, 6 units. .'. Consonants will remain together in 60 x 60 or
To form a triangle, 3 lines can be selected out of 5 3600 ways.
in 5C3 or 10. .' The required number of ways that consonants may
But the length of any two sides of a triangle is not in consecutive positions is--21720n'1 - 3600
greater than the third. :273600
So, in the combinations (2,3,5), (2,3, 6) and Team can be formed as follows:
(2,4,6) triangle.cannot b" fomU, : . mechanical (5) electrical (4)
.'. Total number of ways = 10 - 3 : 7 (a) I 3

(b)
! Ciurn digits 2,3, 4, 5,6 2 2

3 digits out of 5 can be arranged with repeatition in (c) 3 I

53 or 125 ways. wavs :


a) Number of ways 'C, x "Ct : 20
acr= 69
Again, without repeatition 3 digits out of 5 can be (b) NumUer of ways --tcrx
ananged io
tP, or 60 we!$r : ,,i , (c) Number of ways tC, 4Cr :40
:
"
.'. Total numbers consisting of two or more of the .'. Total numberof ways:20* 60 +40:120
same digits is : 125 - 60 = 65. ,' Creative Ouestion-20
$::!'l
Creative Our I The word 'Director' consists of 7 letters.
(a) A man has 6 friends. In how many ways can
In Brothers 6engineering' company, 5
he invite one or more of his friends?
mechanical engineers and 4 etectrig engineers
:)' (b) Find the number of ways in which the letteis
are acting of the quoted word may be rearranged without
(a) There are 4 candidatel ior a p0st and one is
changing the order of the vowels.
to be elected by the votes of 7'il;i. Iii how many
(c) How many different words can be formed
ways can the votes be given? ' '.: from the letters of the quoted word taken 3 at a
(b) Find the number of arlllgemgnts that can be
time?
made of the letters of the qu"otgd word so that the Solution:I for each friend there are two options:
consonantS may not be in'bonsecutive positions Inviting him or not inviting him.
in any of them. .'.Options for 6 friends is:2x2x2x2x2x2:26
(c) In how many ways.ccri"{o eoftnittee of 4 But among these options, I option is such that it
26
engineers can be formed-rcUnsirffig of,at least I excludes all the friends which is not acceptable.
mechanical engine€r ad;l dleffirlc eag*noer. .'. Required number of ways :26 -t '
Chapter-S : Permutations and Combinations 3r5

I The word 'Director' consists of 8 letters of Explanation: Considering 3 vowels as I letter,


which 2 are or's and remaining are different. number of letters is 6 and can be arranged in 6! or
It consists of 3 vowels and 5 consonants. 720 ways.
Since the vowels will remain in same order, let us Again, 3 vowels can be arranged among themselves
consider that they are same letters. ?t
So out of 8 letters 3 are same and 2 'r's.
i" Ztor
3 ways.

.'. The number of arrangements ir = # : 3360 .'. Total number of permutations :720 x3 : 2160
Ans: (b)
The number of ways in which the letters of the
! fne word 'Director' consists of 8 letters of
word 'DIFFERENT' may be rearranged without
which 2 are'r's and remaining are different.
changing the positions of any vowel [DjB.17l is-
Lettersare: rr Diecto
Combination and permutation of 3 letters of this
(a)720 (b) 71e (d) 3se
(c) 360
Explanation: The word'DIFFERENT' consists of 6
word are as follows:
consonants of which 2 are'F's.
Description Combination Permutation
Without changing the positions of any vowel, the
(a) 2 alike, I xocr 1 x6}t x*
I different i": zll:ry= 360 ways.
consonants can be arranged
,Cr
(b) All 3 are 'c, x 3!
.'. Number of rearrangement is : 360 - 1 :359
different
Ans: (d)
.'. Total number of permutations/words
How mAny odd numbers of 4 significant digits can
- ?t
:IxuC,*O+7C3x3t be formed with the digits 2r3,41 5 using each digit
: l8 + 210:228 only once in a number?
(a) 6 (b) 12 (c) 2a (d) 48
Explanation: Keeping 3 in unit place, remaining
letters can be arranged to form odd numbers in 3! or
Simple multiple cholce qi estions:
6 ways.
1. 0!=? IBUTEX. tr-t2l Similarly keeping 5 in the unit place number of
(a)-* (b)* (c)0 (d)l
ways is 3! or 6.
Explanation: We know, n! : n(n - l)! :6 + 6:
.'. Total 12 ways.
Puttingn: I weget, l! : I x 0! or, I : I x0! Ans: (b)
.'.0! :l
In how many ways can a person give computer
Ans: (d)
,l password of 6 letters all of which have 'M' at the
How many words can be formed using the letters
beginning?
of the word BANANA? [DB. 171 (r) "P, (b) "Pu
(a)720 (b) 120 (c) 60 (d) 6
(.)'uP, (d)'uPu
Explanation: The word 'BANANA' consists of 6
letters of which 3 are 'A' s and 2 are N' s.
Explanation: , Keeping 'M' at the beginning, 5

more leffers out of remaining 25 leffers can be


: :
.'. Number of words
*^ ffi: uo arranged in
25P5
ways.

Ans: (c) Ans: (a)


3. In how many ways the letters of the word 7. How many numbers of 5 signilicant digits can be
'ELEPHANT' be arranged so that the vowels formed with the digits 31 4,5,6, 7 using each digit
may occupy consecutive positions? only once in a number where 4 and 6 will remain
(a) 720 (b) 2160 in 2nd and 4th placesrespectively? tCB. l7l
(c) 4320 (d) 20r60 (a) 120 (b) 60 (c) 12 (d) 6
316 Chapter-S : Permutations and Combinations

Explanation: Keeping 4 and 6 fxed in 2nd and 4th Mu ltip le c o mp letio n mu ltip le c h o ic e q a estio n s :
places respectively numbers formed by 3 digits is 12. With 7 points of which any three are not
-'P'=3! =6 collinear-
{nsr (d) (i) 2l straight lines can be drawn.
8. If 4 x nCl =
nC2
then what is the value of n? (ii) 35 triangles can be formed.
(a) 4 (b) 5 (iii) form a plane ligure which has 14 diagonals.
(c) 6 (d) e
Which sentences of the above are correct?
Explanation:4 x nCr -nC, (a) i and ii (b) ii and iii
(d) i, ii and iii
=4n:ry =4n*n(n;1)
(c) i and iii
Explanation: Number of straight lines formed by
+8n:n'-n =)n2=9n .'.n:9 t'.'n*0] joining 2 points out of 7 is 7C2
or 21.
Ans: (d)
.'. (i) is correct.
9. The number of diagonals of a polygon with l6 Number of triangles formed by joining 3 points out
sides is- IDJB. t7l of 7 is 7C3 or 35.
(a) 240 @) 224 (c) r20 (d) 104
.'. (ii) is correct.
Explanation: 2 points can be selected-out of 16
Number of diagonals of a plane figure with 7
points to form a strailht line is -'ucr: 120
But the 16 sides are not diagonals. vertices is: no. of straight lines - no. of sides.
.'. Total number of diagonals: 120 - 16 = 104 :21 -7:14
Ans: (d) .'. (iii) is correct.
10. In how many lvays 6 students can De divided into Ans: (d)
two equal groups? lcB. l7l 13. All the letters of the word 'FATHER' can be-
(a) l0 (b) 20 (c) 60 (d) 120
(i) arranged in 6! ways.
Explanation: 6 students can be divided into two
(ii) rearranged in 719 ways.
equal groups of 3 students in
(iii) selected in 1 ways.
.61 =_lL__ l0 ways.
=3!3121 6x6x2 Which sentences of the above are correct?
(a) i and ii (b) ii and iii
Ans: (a)
(c) i and iii (d) i, ii and iii
11. In permutation and combinations- [DB. 171
Explanation: The word 'FATHER' consists of 6
(i) "c, = ncn -, (ii) "C, * nCr-! - n*'C,
letters all of which are different.
(iii) r! x nC, = nP, .'. Number of arrangement: 6!
\Uhich sentences of the above are correct?
.'. (i) is correct.
(a) i and ii (b) i and iii
The word'FATHER' itself an arrangement.
(c) ii and iii (d) i, ii and iii
.'. Number ofrearrangement = 6! - l:720 - l:719
rn! .t
Explanation:'Cr: r (n
r! .'. (ii) is correct.
- r)!
:6Co: I
n! Number of combinations
nCn
(n-r)! (n-n+r)! ---U-:
nrn
.'. -, (n - r)! r!
vr .'. (iii) is correct.
.'. (i) is correct. Ans: (d)
nC,
* nCr- r - n*'C, which is a formula. 14. The number of ways thAt the faces of a cube can
.'. (iD is correct. be colordd by six different colors is- [BB. l7l
(i) tP, 5c5
r! xoc,=rr *ffi:#r)r=T, , (ii) 6!
Which sentences of the above are correct?
(iii)

.'. (iii) is correct. (a)'i and ii (b) i and iii


Ans: (d) (c) ii and iii (d) i, ii and iii
Chapter-S: Permutations and Combinations 3t7
Explanation: 6 sides can be colored by 6 different Situatlon sA multiple cholce questions:
colors in
6P6
= 6! ways. Answer the quegtions 17 and l8 based on the
.'. (i) and (ii) are correct. following stemr
Ans: (a) nC7=xandnCz=y
15. From a team ol 14 members- 17, If y = 10 then which one is the value of n?
tsCro
(i) 11 can be selected with a fixed captein in (a) 5 (b) l0
ways. (c) 15 (d) 20
raCro x
(ii) l0 can be arrangett in a line in l0! 18. If x : y then what is the value of
nC6
?
ways. (a) e (b) 36
(iii) one or more can be invited in 2ra - I ways. (c) 54 (d) 84
Which sentences of the above are correet? oC)
Explanation-l7: =y = l0
(a) i and ii (b) ii and iii
(c) i iii (d) i, ii iii t): l)= ro=nz-n:20
and and
-n(L:
- 2l -tvro -?
=rn(n--
2 -rv
Explanation: Fixing a captain, l0 more members
out of 13 can be selected in
l3C1s
ways. + n, - n - 20 : 0 =+ n2u 5n * 4n -20 = 0
.'. (i) is correct. +(n-5)(n+4):'g
l0 people can be selected out of in taCls ways.
14 .'. n = 5 or n = 4 lnot acceptable]
l0 people can be arranged in a line in l0! ways. ... n:5
.'. Total number of ways = l4Cro x 10! Ans: (a)
.'. (ii) is correct. Explanation-I8: x = y "C, : nC2

One or more can be invited in}ta - I ways.


?
.'. (iii) is correct.
=)n:7 +2.'. n:9
.'. 'Cu =
9Co: 84
Ans: (d)
Ans: (d)
16. With the digits 1,2,3,4,5
(i) 120 numbers can be formed taldng atl digits. Answer the questions 19 and 20 based on the
(ii) 120 numbers of 4 digits can be formed following stem:
The word
(DHAKA'consists of 5 letters of which 2
without repeatition.
(iii) 125 numbers of 3 digits can be formed with are tAts and remaining three are difrerent
repeatition. 19. What is the number of rearrangements of the
Which sentences of the above are cmrect? letters of the word taken all at a time?
(a) i and ii (b) ii and iii (a) 59 (b) 60
(c) i and iii (d) i, ii and iii (c) 119 (d) r2o
Explanation: Given digits l, 2, 3, 4, 5
20. What is the number of combinations of the letters
Total numbers formed 5!
of the word taken 4 ata time?
.'. (i) is correct.
(a) 3 (b) 4
Number of ways that 4 digit numbers can be formed
sPa: 5! = (c) 7 (d) ll
using 5 digits is: 120
.'. (ii) is correct. Explanation-l9: Total number of arrangements is
, 5l
Number of ways that 3 digit numbers can be formed :fr,:60
using 5 digits with repeatition is = 53: 125

.'. (iiD is correct. .'. Number of rearrangements is:60 - I ;59


Ans: (d) Ans: (a)
318 Chapter-5 : Permutations and Combinations

Explanation--20: The w-ed,.'6,DHAl(l{' consists of 5 Explanation: 2nP3 :2 x nPa

letters. ,: or,2n(2n - 1) (2n - 2):2 x n(n - l) (n - 2) (n -3)


Out'of 4 different letters 4 can.be sg.lwted in aCa
or or, 4n(2n - l) (n - 1) : 2n(n - l) (n - 2) (n - 3)
I way. .. ii,. or,2(2n - l): (n -2) (n -3)
Taking 2' A's, 2 more"lgtg*q c41t.F.selected out of or,4n -2- n'- 3n -2n+ 6 .

3C2
remaining 3 different letters in or 3 ways.
.'. Total number o combinations: I + 3:4 or, (n l) (n - 8):0
-
Ans: (d) .'. n: l, 8
Multiple choice questions of different universities But n: I is not acceptable.
' :.n:8
21. = 5 then whieh me*r:&G*talrre of n? Ans: (d)
"ffi; 24. The number of arrangements of the word
IRUET.09-101 rADMISSION'taking all the letters is-
(a) o (b) 4
' [RU: 15-16; KU: 14-151
(c) s (d)6

Explanation: Given3fr5= 5
@# (b) e!

(c)? (d) None of these


l) (n - 2)!
or'Jl6lf
-
n(n :5 Explanation: The word'ADMISSION' consists
g letters of which 2 are 'I's and 2 'S's.
of

or, n(n - l):30 :#


.'. Number of permutations
or,n2-n-30=0
or, n'- 6n + 5n - 30:0 Ans: (a)
25. In how many ways the letters of the word
or, (n - 6) (n + 5):6 'BANGLADESH' can be arranged without
.'. n : 6 or n - - 5 [ not acceptable] .changing the position of the part'DESH'?

Ans: (d) [JNU: 16-17l


(a) 720 (b) 360
(c) 2520 (d) 60480
[JU: 09**O,HU: #0q CU: 03-041
(a)Eor9 (b),&or'10 i: Explanation: Since the part 'DESH' will not
change its position, so it is out of consideration.
(c)7or8 '(d).7 or.l0.
Remaining part 'BANGLA' consists of 6 letters'bf
Explanation: nPa: 14 x
n-2Pr
which 2 are "A's.
or, n(n - l) (n -2) (n - 3) : l4(n - 2).(n -3) (n -a)
or, n(n - l): la (n -a) .'. Number of permutations: ff : :60
or,n'-n- 14n-56,:, i .::.',,
Ans: (b)
or, n'- l5n + 56 = o 26. rn how ways can 5 people
or,(n-7)(n-8):0 :rn, ffi'*fi:;,,
.'.n:7,8 .,....,;.. @) 2a (b) r20
Ans: (c) ' (c)720 (d) 5040
If2'P3 :2 x nPa
then n = ? [JU:13-l4l Explanation: 5 peopte can be arranged in a line in
(a)2 (b) 4 5! or 120 w.ays.
(c) 6 (d) 8 Ans: (b)
Chapter-S : Permutations and Combinations 3t9
27. If the permutations of the letters of the word Keeping '[J' at the beginning and the end,

'RAJSHAHI' is k times the permutations of the


letters of the word 'BARISAL' then what is the
remaining 6 letters can be arranged ,"#.: l8o

value of k? lDU. 17-181 ways.


(a) 2 (b) 3
(c) 4 (d) s Ans: (b)
Explanation: The word 'BARISAL' consists of 7 30. How many ways the letters of the word
letters of which 2 are 'A's and remaining are 'Permutation' may be rearranged without
different. changing the positions of any vowel? [DU: l8-l9l
(a) 360 (b) 460
.'. Total number of arangements -+, :2520.
(c) 459 (d) 3se
The word 'RAJSHAHI' consists of 8 letters of Explanation: The word 'Permutation' contains 5
which 2 ate 'A's, 2 'H's and remaining are vowels (e, u, a, i, o) and 6 consonants (P, r, m, t, t, n).
different. Without changing the positions of any vowel, the
.'. Total number of arrangements :#: 10080. consonants can be arranged among themselves in
6t
Here 10080 =4x2520 ways.
,I
Here, 10080 : 4 x2520 .'. k : 4 The word 'Permutation' itself is an arrangement.
Ans: (c)
28, How many arrangements c.an be made out of the Number of rearrangementis :*.- I = 359
letters of the word 'MATHEMATICS' all of Ans: (d)
which have 'T' atthe beginning and the end?
31. How many numbers of 6 digits can be formed with
[DU: t6-t7l the digits 2,1,6,0,5,9? 06-071
[KUET.
(a) 10080 (b) e680
(a) 120 (b)720
(c) 50720 (d) 90720
(c) 600 (d) s+o
Explanation: The word 'MATHEMATICS'
Explanation: Given digits: 2,1,6,0, 5, 9
consists of I I letters of which 2 are 'T's, 2 M's and
6 digits can be arranged to form numbers in 6!
2'A's.
ways.
Keeping 'T' al the beginning and the end,
But 5! of these numbers start with 0.
remaining 9 letters will be arranged.
.'. Meaningful 6 digit numbers:6! - 5! = 600
Number of permutations *#. Ans: (c)
9x8 x7 x 6 x 5 x 4 x 3 x 2 x I :90720 32. How many different arrangements can be made
2xlx2xl of all the letters of the word 'ENGINEERING'
Ans: (d) rvhere the three 'e's stand together? lDU. l0-l I I
29. How many arrangements can be made out of the (a) tstz (b) 16s0 (c) r5r20 (d)277200
letters of the the word 'CALCULUS'all of which 9!
:
Explanatiotz 15120
have'U' at the beginning and at the end? 1ffii
[JNU: 15-16, ll-12; JU: 15-16; Ans: (c)
DU: 1l-12; RU: 08-09; SUST: 04-051 33. In how many ways 3 boys and 4 girls can be
(a) e0 (b) 18O. trranged in a row that the boys will remain
(c) 280 (d) 360 toget[er? [DU:99-001
Explanation: The word 'CALCULUS' consists of 8 (a)720 (b) 24s0
letters of which 2 are'C's,2'L's and 2 'Uls. (c) 4320 (d) 144
320 Chapter-5 : Permutations and Combinations

Explanation: Since 3 boys will remain together, 36. In how many ways can 5 books of poetry, 3
considering them ffi l, number of people is 5 and books ofstory and 2 books of novel be arranged
cap be arranged in 5! ways. on a shelf so that books on the same topics may
' Again, 3 boys can be arranged among themselves in be together? lRU. 17-lEl
(a) 450 (b) 3e60
3! ways.
:720 (c) 8640 (d) none ofthese
.'. Total number of permutations is = 5! x 3l
Explanation: Considering 5 books of poetry as I
Ans: (a)
gfoup, 3 books of story as another group and 2
34. The number of arrangements of the word books of novel as third group; 3 groups can be
'SCIENCB'where the vowels occur together is- arranged in 3! :6 ways.
[DU: 97-9Ei IU: l4-l5l Again,
(a) 60 (b) 120 5 books ofpoety can be arranged in 5! : 120 ways.

(c) 180 (d) 420 3 books of story can be arranged in 3! = 6 ways.

Explanation: The word 'SCIENCE' consists of 7 2 books ofnovel can be arranged in}l= 2 ways.

letters of which 2 are Cand 3 vowels of which 2 areB. .'. Total number of arrangements : 120 x 6 x2 x 6
:8640
Since the vowels will remain together, considering
Ans: (c)
them as I letter number of letters is 5 and can be
<t
37. In how many ways can 6 boys seat in a bench of
arranged in ways. 3 seats?
ii [RUET.09-l0l
(a) 3! (b) 6!
But the vowels can be arranged among themselves
(c) uc, (d) uP,
m,3! ways. Explanation: 6 boys can be arranged in 3 positions
6P3
in ways.
: * = r to
Total number of permutations
lf lf Ans: (d)
38. How many"4 digit numbers can be formed with
Ans: (c)
the digits 2,3, 4, 5, 6,7 and 8 taking each digit
35. The number of arrangements of the word only once in a number?
'daughter' where the vowels wilt not occur [JU: 12-13; RU: 07-08; KU: 03-041
together is- l.II.]: 11-12; PUST: ll-l2l (a) 28 (b) 780
(a) 36000 (b) 360200 (c) 840 (d) 480
(c) 360800 (d) 360e00 Explanation: Total numbers is: 7P+: 840
Explanation: The word 'daughter' consists of 8 Ans: (c)
letters of which 5 are consonants and 3 vowels. 39. How many odd numbers of 3 significant digits
.'. Total no. of permutations = 8! : 40320
can be formed with the digits 41 51 61 7 and 8?
IKUET. ls-161
Considering 3 vowels as I letter, number of letters l"'.):" (b)25
is 6 and number of permutations is 6!
{il24
(c) s0 (d) 36
Again, the vowels can be arranged among Explanation: Keeping 5 in unit place 3 digit
themselves in 3! ways. numbers can be formed in
aP2
ways.
So the number of permutations that vowels will Again, keeping 7 in unit place 3 digit numbers can
occur together is : 6! x 3! 'be formed in aP2 ways.

.'. Required no. of permutations = 40320 - (6! x 3!) .'. Number of odd numbers of 3 significant digits is
:36000 : 2 xapr:24
Ans: (a) Ans: (a)
Chapter-S : Permutations and Combinations 321
40. How many 3 digit,numbers Sdatcr &a* 200 can 44. Horv many rearrangements is possible with the
be fonned,usingthedigits,t 3r.5;7+t$i ,, , letters of the word 'TIME' that starts with a
[CU: 12-13; DU:9G971 vowel? [DU: 98-991
(a)24' (b)48 i :.; (d) 6 (b)24
(c)
\!, 60
vv (d) 64
vT
\s,
i , (c) 1l (d) 12
Explanation: Sin93 the nua!gry,qlg.9{;$are of', 3 r;_I_
..rit. disits and
:._, ,1,ii:It .:i.i^ t; r - Explanation; The word'TIME' consists of 4 letters
greater than 200, these must start wi6 3'oi5 or 7 or 9.
nPi of which 2"are'vowels and 2 consonants.
So the l't position can be filled in dr 4 ways.
Again, remaining 2 positions cem h filled by
In I't position, I vowel can be arranged in 2P1 or
remaining 4 digits in aP2 or 12 ways. 2 ways;
3P3
.'. Required nunrber of permutatiotts,+14rx-12 : 48 Remaining 3 letters can be arranged in = 6 ways.

Ans: (b) 1.
.'. No. ofarrangements is = 2 x 6 = 12
41. How many 3 digit numbers lylu:be#veen 100 Since the word 'TIME' is rrct start with a vowel, the
and 500 carr be formed wit[:the dtsie 1r 2,3, 4, no. of rearrangement is also 12.
5,6,7 without repetition of any digit? Ans: (d)
F14l 45. How.many numbers greater than 4000 can be
(a) 60 (b) r2o G) 2lF (d) 24iry formed with the digits 0, 3, 5, 6, 8 without
Explanation: 4 x6Pr: 129
repetition of any digit in any number?

42. How many arrangements can be midb'iirit of the IBUET. tt-t2l


letters of the word 'courage' all of whiGh have a @) Aa (b) 168 (c) 1e2 (d) r78
vowel at the beginning? Explanation: 0, 3,5,6, 8
[DU;14-151
(a) 180 @)na Total numbers
(c) 2880 (d) 5M0 = ( 4 digit numbers + 5 digit numbers)
Explanation: The yvord 'COURAGEI consists of 7 :3xoPr+4xaPo:168
letters of which 4 are vowelsand 3 consonants.
Ans: (b)
All the letters are different.
"At 46. In how many ways the letters of the word
the I't position I vowel out of'4 r*'br arranged
in
aP1
or 4 ways 'Institute' be arranged so that the vowels may
Remaining 6 positions can be filled by remaining 6 occupy even positions? IKUET.09-101
letters in
6P6
or 720ways. @) 240 (b) 280
Total number of permutations
.'. : 4'i 12b' : 2880 (c) 380 (d) 440
Ans: (c) Explanation: The word 'Institute' consists 9 letters
43. How many arrangements catt be madc out of the of which 4 are vorvels (I, i, u, e) and 5 are
letters of the word 'courage' alt of wffih have a consonants (n, s, t, t, t).
consonant at the beginning? tBtIEA l244l The positions that are required to be filled are:
(a) 720 (b) 1260
I, II, III, IV, V, VI, VII, VNI, Ix
(c) 2106 (d) ?160 : ,.
4 even positions can be filled by 4 vowels in 4!
Explanation: The word 'Courage' consiqts of 7
ways.
letters of which 4 are vowels and 3 are consonants.
So the I't position can be filled by a consonant in Remaining 5 positions can be filled by 5 consonants
"ri
'P, ways.
st
,n,JT ways.
Remaining 6 letters can be .**ii6A inlrernaining 6
positions in 6! ways.
.'.Totalhumber of ways :41 xfi :UO
Total ways -'P, x 6t = 2160 ::

Ans: (d) Ans: (d)


3;22 Chapter-S,:, Permutations and Combinations

47; 'In hw .many ways 'tiro'iletters of tho word 50. ,flow many 3 digit numbers ean be formed using
tequation' can i bG' armngsd so that the the digits lr2r 41 6, 8, 9 with repetition?
cqnsonants occupy the odd positions? IDII 99-00; RU: 06-071
. lJ i [JNU: 08-09,05-06; (a) 120 (b) 216
.JIJI06-07; KU: 07-08; RU: 05-061 (c) 2 (d) 60
(a) 2080 .,(b) 2880 . Explanation: Here, total digits:6
(c) 1090 @r?rtfi J' n: 6rr:3
, Explanation:r The wgrd :equ*foml ,eonsists of 8 .'. Total number of permutations : n'= 63 = 216
letters of which 3 are consonants and 5 vowels.
Ans: (b)
The positions,that are reWird to be #illed are: 51. The telephones of 'Khulna' are of 5 digits and
o@@@ @@@,@,, ,,,:: start with 72, 73 or 74. How many telephone
' 3 consonants can be ananged in 4 odd positions in
aP3ways. connections can be given in Khulna city if the
i.r::- . : ,:r.
telephones of Khulna are of 5 digits?
Remaining 5 positions can be filled by,5 vowels in
IKUET. l0-lll
5! ways. , i
(a)loa (b) to6
.'. Number of permutatiog-ls.,i{ : tF; x 5! =,2880
(c)3xloa (d)3x106
Ans: (b)
Explanation: First 2 positions are fixed for code
48. In how many lvays can 5 students of Chemistry
numbers and can be filled by 72, 73 or 74 in 3
dept. .and 5 students of Pharqacy Dept. sit at a
ways.
round table so that no two Pharmacy Dept.
i'
Remaining 4 digits can be filled by l0 mathematical
students may be side by side?
digits with repetition in l0aways.
[RU: 15-16; RU:08-091
Total number of connections = 3 x lOa
(a) 2880 (b) 2840
Ans: (c)
(c) 2480 (d) 2440 . 52. Afsana forgot last three digits of her mother's
Explanation: Fixing I student b'f Chemistry,
remaining 4 canbe arranged ini4! ways. mobile number. How many times she should try
There are 5 positions betweenlihese 5 Chemistry to get connection? [RU. 17-l8l
students that can be' filtsd by 5' Pharmacy (a) 1000 (b) 2000
students in 5P5
= 5! ways (c) 3000 (d) 4000
.'. Total number of permutations:4! x 5! : 2880 Explanation: Total mathematical orgits = l0
Ans: (a) ': The number of ways to form last three digits of
49. In how many ways can S,dfffeient pearls be mobile number using l0 digits is : n'= 103 = 1000

strung on a band so as' to .forfr, a, necklace? .'. 1000 times.


p{mT. 1n-1.;,l Ans: (a)
53. How many three digit numbers formed by the
(a) 7! (b) s! G\t' G)# digits 1,2,0 is divisibleby 2? [DU: 16-17l
Explanatiron: Fixing 'l pearl, remaining 7 pearls (a) 6 (b) 18
,,-. : r.,
can be alranged in 7! ways., (c) 4 (d) 12
Again for clockwise and for anti'clockwise Explanation: Given digits l, 2, 0
pennutations, same necklaee is formed. Numbers divisible by 2 wilt consist 2 or 0 in the
qrit place
.'.Number of *uy, : $ .

Keeping 2 in the unit place, given digits can"be


Ans: (c) arranged with repetition in 32 or 9 ways.
Chapter-5 : Permutations and Combinations 323

Out of these 9 numbers, 3 numbers have 0 at the Explanation: We know, the formula for
beginning which are not meaningful 3 digit combination: nCr+ncr-.I :n*tC,
numbers.. . Ans: (c)
Keeping 2 in the unit place the number of three If loCr + ? = t'C, then what will set,,in the '?'
digit even numbers is = 9 -3 : 6 marked place? UU. 17-181
Similarly, keeping 0 in the unit place the number of (a) "c, (b)'oco
three digit even numbers is:6 (.) "c, (d) "cn
.'. Required total numbers is : 6 + 6: 12 Explanation: Formula: nC,
X
nCr-r
- n*tC,

Ans: (d) .'. 'oc, + locr*r - 1o+lg,


1 * loCz: llCr
If nC2 = i * nCa
then what is the value of n? =+ 'oC,
Ans: (a)
lRU. 17-181 59. How many triangles can be formed by joining
(a) 3 (b) s the angular points of a plane figure of 16 sides?
(c) 7 (d) e [JU: l4-151
nC2 :? ."a, (a) 560 (b) 6s0
Explanation:
(d) 460
;
(c) 600
*n(n- 1)__1 n(n- l\(n-2)(n-3) Explanation: Required no. of triangles: 'uCr:560
2t 5' 4l Ans: (a)
_1_3.(n-2)(n-3) 60. How many triangles can be formed with the
=r: s"-zq straight lines of length 1,2,3,4 unit?
= (n - 2) (n - 3):20 IBUET. 13-141
=n'-3n-2n+6-20:o (a) I (b) 2
=>n2-5n-14:o (c) 3 (d) 4
=+n(n -7)+2(n- 7):0 Explanation: 3 sraight lines out of 4 can bp
aC3
selected in or 4 ways.
=+(n-7)(n+2)-g
.'. n - 7,1: -2 (not acceptable; Butl+2:3, l+2<4,1+3:4
Ans: (c) In the above three cases triangle cannot be
nCs nCz formed.
If = then n =? UNU: 16-17l
(a)2 (b) 7 .'. Number oftriangles : 4- 3: I

(c) 12 (d) 3s Ans: (a)


Explanation: Since nCs: nCz 61. How many diagonals can be formed by joining
.'.n-5+7:12 the angular points of a plane figure of I0 sides?
Ans: (c) lRU. 08-0el
nC12 nCE 22Cn? (a) 10 (b) 20
If = then which one is the value of
(c) 35 (d) 4s
IKUET. 07-081
Explanation: Number of diagonals :'oC, - l0 : 35
(a) 331 (b) 332
(c) 231 (d) 232 Ans: (c)
Explanation: Given, nCrz : nC8 62. In world cup football32 teams play in 4 groups.
How many games will take place in first round?
.'. n- 12+8:20 (b) 32
@) 2a
Hence "Cn:"Cro :231 (c) a8 (d) e6
Ans: (c)
57. Which one is the value of nCr.+ 'Cr-, ? Explanation: Nurnber of teams in each goup :ff: O

IKUET. tt-tz,BUTEX. tt-121 Number of games among 4 teams is : aCz I6


(a) n*'c,-, (b) n-'c*, .'. Total numberof games in 8 groups is:6 x 8 :48
(.) n*'c, (d) n-tc, Ans: (c)
324 Chapter-S : Permutations and Combinations

63. After a meeting atl the the'peo$,st&nded there 66. 5 childrcn are needed,to select from a group of 6
handshaked with onsi another.', ff, the total boys and 4 girls to admit a course. In how many
ways the selection process can be done consisting
number of handshakes is 66, then how many
of just 2 girls? IBUET. 08-091
(a) ll0 (b) 120 (c) 125 (d) 130
(") r'l' (b) 12 (c)24:''"";' (d) 33 Explanation: 2 girls out of 4 can be selected in
aCz

Explanation: Letrthe number of people-b..e 'n'.


So
nC,
- 66 [A handshake$pedS,Bpsde]
;tt'"[h of 6 can be selected h 6c3
or 20 ways
.'. Total number of ways :6 x20: 120.
or, &i! :66 or,n2 - -
n 132'= o Ans: (b)
67, In how many: ways can a team be formed
or, (n - 12) (n + I l):0 ,
consisting of 3 boys and 2 girls from a group of 6
.'. - 12, n'= -l I ltndt 6to@bleJ ''
Jl ' 1
boysand 5 girls? lDU.l+-16l
/ '; ". (a) l0 (b) 20
:. n: 12
:.

(c) 50 (d) 200


Ans: (b) Explanation: 3 boys out of 6 can be selected in
6Cr '

64. In how many selections of 12 books tqken 5 at a ways.


5C2
time will2 particutar books'notgecur? . 2 girls out of 5 can be selected in ways.
6C3 sCz:
.'. Total number ofways = x 20 x l0 = 200
[DU: 04-0$;'ftU: 14-151
Ans: (d)
(a)I20 (h)2!5
68. In how many different ways can 2 boys and 2
(c)252 (d) I28 girls be selected out of 10 boys and 8 girls?
Explanation: Since 2 particular books will not [JU: 12-13; DU: 95-961
occur. (a) 73 (b) 300
(d) 1260
.'. No. of selections : -'c, -'oC, 2i2
l' (c) 51
= Explanation: Required no. of ways - 'oC, x
8Cz
Ans: (c) = 1260
65. In how many ways -can e '4 members sub- Ans: (d)
committee be formed conslstirg at least 1 69. In how many ways can a sub-committee be
gentleman from a board consisting of 6 formed consisting of 2 gentlemen or 2 ladies
' ii from a school management committee consisting
gentleman and 4 tadies? of 4 gentlemen and 3 ladies? UU: 12-131
. [CUi 12-13; DU: o2-031 (a) 6 (b) 3 (c) e (d) 12
(a) 504 (b) 210 Explanation: Required no. of ways - *Cr,*3Cz: 9
(c\ 126 (d) 84 Ans: (c)
Explanation: Possible ways of forming. committee
70. In how many ways can a committee be formed
consisting of 5 members from a group of 6 boys
is as follows: and 5 girls? lDU. 0e-101
Ways Gentleman (6) Liidy (4) @) A4 (b) 360 (c) 455 (d) 720
(i) 4 0 Explanation:
Boys (6) Girls (5)
(ii) !
3
(a) I 4
(iii) 2 2._ (b) 2 3
(iv) I 3:. (c) 3' 2
.'. Totalnumberof ways -, ......r,, ,:.r., ,
(d) 4 I
.'. Number of commiffee
- tc, x 4co qsczx 4cr + 5c; *\cr.' sco x 4cr =
6C)1
x 5Cl q6Czx 5C: + uC, x 5Cz + uco x 5Cr
:10+40+30 +4:84 , =6 x 5 + 15 xlO+20 x l0+ 15 x 5 =455
Ans: (d) Ans: (c)
325
|
Chapter-S : Pdrmutatioris and Combinations

71. How many different woidi cin, be f-gimed from More daltiple choice gaelionq of different
the letters of the word 'AMERICAi'tilien three universltles admtsslon:
at a time?
-lt" [.Hhi{Sl6; CU:02-031 75. The number :of arrtngements of the word
(a) 125 '(b) 135 ; 'KACHUA' where the voweh occur together is:-
(c)140 ' (d) 11J.^" ., ,
. :
[DU: l2-t3l
Exptanation: The *ora iAlififjbe\licrir;iists of z
(a)24 (b)72 (c) 1a4 (d) 8 , :

letters of which 2 ara'A's.


Explanation: The word 'KACHUA' consists of 6
Lettersare:AAMERIC
letters of which 3 are vowels and 2 A's.
Cqabention Permutation
Considering 3 vowels as I letter, number of letters
(i)2sarne&l {'.1,:.ttG

different
5 xfi: 15 is 4 and can be arranged in = 4! ways.
g+:i-:.:'rrr .-"-
Again, the 3 vowels can be arranged among
(ii) all different "Cr :20 20 x 3! =120 ?f
themselves in ways.
.'. Total number of wor.,{s = li + l2.Se .l}5 ,
ii
Ans: (b)
.'. Total number of permutations : 4t x#. : r,
72. A man has 3, 4 and 5 notes of tlc 500, 50 and 5
respectively. In how niany ways he can pay for Ans: (b)
the different products by ndt'mutiiig tn'e change 76. How many four digit numbers greater than 7000
of the money? [K[IET: 17-181, can be formed with the digits 3, 5r 7 ,8, 9 without
(a) 59 o) 60 repetition of any digit in any nurnber?
(c) 61 (it) 119 i JDU: 03-041
Explanation: No. of ways = (3 t l) (4,+ l),(5+ 1) - I (a)27 (b) 81
:4x5x6-l:l19 (c) 72 (d) s6
Ans: (d) ." .; -i i:; i tl.' Explanation: Given digits 3, 5,7 ,8,9
73. Number of factors of the number 277200 is- Here 4 digit numbers that greater than 7000 must
lru. 17-l8l start with 7,8 or 9.
(a) 120 (b) 160 Number of permutations :3 x 4Pf 72
(c) 180 (d)200 !
Ans: (c)
Explanation: 277200 : 2772 x 100 77. A committee of 6 members will be formed from a
:2x2x3 x 3'x 7 xll x2 x 50 bo.ard of 7 Englishmen and 4 Americans. In how
:2xZx3x 3 x 7 x 1l x2x2 x 5 X5' many ways the committee can be formed so as to
:24x32x52x7lxlll include at least 2 Americans? [DU: 0l-021
.'. Number of factors :
(4 + l)Q'+ I)fr.+ l).22: 180 (a) 350 (b) 371
Ans: (c) ii' -..'!i
(c) 381 (d) 41s
74. There are 4 letters and, 4 ellyelops wilh specific Explanation: Possible ways of forming committee
addresses. In how many ways each of the 4 is as follorvs:
letters can be entered into the envelops gf wrong English (7) American (4)
addresses? lRU. 16-17l
(i) 4 2
(a) 6 (b) E
(c) e (d) (ir) 3 3
12
Explanation: Out of 4 lctters't,;lffibr **ill be in (iiD 2 4

correct address and 3 othee **,$rong: address. .'. Total number of ways
Similarly for other three letters. -*'co x 4c2.. 7cs * 4cr +?cz x 4ca
.'. Required number of ways'= g t'g'i'3 + 3 '= 12 :2lj + 140 + 2l :371
Ans: (d) . ,. Ans: (b)
326 Chapter- 6: Trigonometric Ratios

Chapter-6: Trieonometric Ratios oDit :(# " *) degree


radian

= (ff) degree :T degree = 56o15'


1. Express in radian:
(i) 18" 33' 45" (ii) 45" 12' 23.2" $ory radian : (# -ry)degree
(iii) 73'7'30!'

(i) 45' : (t*)


:I(tgorz
x 4il
Solution: = /degree
#
= (15 x47) degree = 705o

'. 33'4s": (i, i)' : ('-09' , "


" '
(i) If D and R are the measures of an angle in

- r\+ * ool - r1\'


r3s_\'_ sexagesimal and circular uni! show tfr"t
_
\tol
ft =
f
So, 18o33'45" :(tt *)" : Solution: 180o: n radian .'. Do:# * ,
ffu)"
: nD
:W,.#) radian =#radian lffi radian
But the angle in radian: R
(ii) 45"12'23.2"
. rD _D_ D
= 45o + 12' +23.2" ..1g0 -r\-?lg0 _R
-n
= 45o. (#)" + (-23'2-60)' (ii) If x and zare the measures of an angle in

:45..(+)'.(ffi) degree and radian then show tnat,ft :ff


: 45o + 0.2" + (0.0064444444) Solution: l80o:mradian.'. xo:# "*
: 45.206444444. radian fix
# :fE6 radian
:0.25715r radian
. 7rx- _ n_7[x _22 x
(iii)30,=(#) :# lg0 "- go - zL.. g0- n
(iii) The sum and difference of two angles are
7'30" =
Q )', = (fl',: (*-J : (*)" Iradian and 10 respectively. Find the smallest
angle in radian.
:. 7 3o7'30" : (#)' Solution: Let, the angles are x and y radian.
Qr'J":
: (ft. f)" radian :# radian
And the smaller angle is y.

By given condition, x + y : I and x-y : l' :#


2. Express in degree:

(D+ radian GD* radian Now,x+y-x*y:l-#


$iD# radian
+
(' #) radian

(iv) Divide the angle 54o in three parts such that


sotution: (D# radian:#,.#) degree
the radian measure of first angle exceeds that.of
:[/tso15x z\ sOcond by
# and the sum of the second and
Jdegree
- (12 x 7) degree = 84o third rrgi. is27o.

You might also like